v3 block 3

Lakukan tugas rumah & ujian kamu dengan baik sekarang menggunakan Quizwiz!

A man has had 4 diagnosed sinus infections in the last 5 months, so you order a CT scan of his sinuses. He does not believe that any of the episodes completely resolved. The patient has a long history of excessive seasonal allergies; he treats them with daily oral OTC anti-histamines and he has repeatedly declined beginning allergy immunotherapy. He also has a history of asthma that is well controlled with daily inhaled corticosteroids. Question What imaging discovery do you expect to find? 1 Hypodense mass 2 Mucosal thinning 3 Opacification 4 Hypoattenuation 5 Translucency

Answer: 3 opacification patient is most likely suffering from chronic rhinosinusitis (CRS); there may or may not be involvement of nasal polyps. The CRS would be classified as chronic due to the fact that he has had symptoms for over 12 weeks. CRS without nasal polyps accounts for up to 65% of cases of CRS. Risk factors that contribute to this condition include the presence of allergic rhinitis and asthma. On computed tomography (CT) studies, patients suffering from CRS without nasal polyps will generally present with sinus opacification (or sinus ostial obstruction) as well as mucosal thickening of the affected sinus cavity. Mucosal thinning and translucency are not associated findings on CT in CRS. Hypoattenuation generally describes an area whiter than usual and is used when referring to organs, such as the liver or kidneys.

48-year-old man presents with a 2-day history of left-sided groin and scrotal pain. He has had similar pain episodically for several months, but it has recently become much worse after a weekend of helping his brother move furniture. He admits that he is not in good physical shape, and he thinks he may have pulled a groin muscle. He is in a monogamous relationship with his wife of 17 years. He has never had any testicular or scrotal conditions, and he has a negative surgical history. He denies fever and urinary symptoms. He has no allergies and takes no other medications. On physical exam, the patient has normal sexual development, with no edema, warmth or erythema present in the scrotum. No skin lesions are present. On palpation, there is mild tenderness on the left scrotum. However, with Valsalva, a small bulge is palpable in the left scrotum, and the patient's reported pain level increases. When he lies supine, the bulge is no longer palpable. Question What intervention is most appropriate for this patient's suspected condition? 1 Ceftriaxone plus doxycycline 2 Manual detorsion of testicle 3 Rest with elevation of the scrotum 4 Surgical referral 5 Weight training for improved strength

Answer: 4 surgical referral This patient's presentation is consistent with an inguinal hernia. For patients who are symptomatic or who have very large and/or recurrent hernias or hernias with incarceration, surgical referral is recommended. Ceftriaxone plus doxycycline is an appropriate intervention if a patient's scrotal pain is due to epididymitis. However, if the patient had epididymitis, his exam would be more likely to demonstrate edema, erythema, warmth, and tenderness not necessarily worsened by Valsalva. Manual detorsion of the testicle can be attempted in a patient with testicular torsion while awaiting surgical intervention. A patient with testicular torsion may present with unilateral scrotal pain, but he would likely also have edema, erythema, as well as even nausea and vomiting. The pain level is so intense that it is unlikely a patient would take 2 days to present. The episodic nature, which worsened with moving furniture (Valsalva), is not associated with torsion either. Rest with elevation of the scrotum can help with acute epididymitis, but it does not have a specific role in treatment of hernia. Weight training for this patient would certainly not help his hernia, and it could aggravate his condition.

A 30-year-old woman with no significant past medical history presents with a history of recurrent palpitations. These episodes occur primarily upon exertion. She recalls periodic bouts of anxiety, panic attacks, and lightheadedness. She denies fever, chills, changes in weight, chest pain, shortness of breath, rashes, diaphoresis, abdominal pain, nausea, and vomiting. She denies any history of cigarette smoking, drug, or alcohol use. Her physical exam revealed normal vital signs. The cardiac exam revealed a high-pitch late systolic click at the apex. The valsalva maneuver and a standing position result in prolongation of the murmur and a movement of the click to earlier in the cardiac cycle. The remainder of her examination is normal. Question What is the most likely diagnosis? 1 patent ductus arteriosus 2 Mitral regurgitation 3 Mitral valve prolapse 4 Mitral stenosis 5 Hypertrophic cardiomyopathy

Answer: 3 Mitral valve prolapse This patient's most likely diagnosis is mitral valve prolapse. The classic auscultatory finding is a mid-to-late systolic click, which is present due to the leaflets prolapsing into the left atrium, resulting in tensing of the mitral valve apparatus. It may or may not be followed by a high-pitched, mid-to-late systolic murmur at the cardiac apex. Activities that reduce end-diastolic volume, such as Valsalva maneuver or a standing position, cause an earlier click and a prolonged murmur. In the supine position, especially with the legs raised for increased venous return, left ventricular diastolic volume is increased, resulting in a click later in systole and a shortened murmur. Patent ductus arteriosus is characterized by an abnormal patency of the ductus arteriosus, which diverts blood from the right side of the heart to the systemic circulation during fetal life. Blood typically shunts from the higher-pressured left side (systemic circulation) to the lower-pressured right side (pulmonary circulation). This murmur is described as a hollow, machinery-like murmur that is continuous throughout the cardiac cycle. Mitral regurgitation is a medium to high-pitched apical, holosystolic, harsh murmur, which may radiate to the left axilla or less commonly to the left sternal border. It may be soft or loud and associated with a decreased S1. The murmur of mitral stenosis is a decrescendo low-pitched rumble usually limited to the apex with an accentuated opening S1 and opening snap that follows S2. The murmur associated with hypertrophic cardiomyopathy is a loud, harsh systolic murmur present along the left sternal border. The gradient and the murmur may be enhanced by maneuvers that decrease ventricular volume, such as an upright posture, standing, or Valsalva maneuver. It is decreased by increasing ventricular volume or vascular resistance, which occurs with squatting, sustained handgrip, lying down, or straight leg raises.

A 70-year-old woman presents with a 3-day history of shortness of breath at rest. She has been finding it difficult to walk short distances due to shortness of breath. Additionally, she is experiencing orthopnea and nocturnal dyspnea. Her past medical history is significant for hypertension, hyperlipidemia, and myocardial infarction. The patient denies cough, fever, chills, diaphoresis, anxiety, chest pain, pleurisy, cough, nausea, abdominal pain, vomiting, diarrhea, rashes, lightheadedness, and syncope. Upon physical examination, the patient is short of breath; she requires numerous pauses during conversation. She is afebrile but tachycardic and diaphoretic; her extremities are cool. The exam reveals a diminished first heart sound, S3 gallop, laterally displaced PMI, bibasilar rales, dullness to percussion, and expiratory wheezing. There is no JVD noted, but 2+ pitting edema of the lower extremities to the level of the mid-calf is evident. Question What is the most likely diagnosis? 1 Congestive heart failure 2 Pulmonary embolism 3 Myocardial infarction 4 Chronic obstructive pulmonary disease 5 Bacterial pneumonia

Answer: 1 CHF This patient's presentation is most consistent with congestive heart failure. Manifestations reflect impaired pumping ability of the heart, reduced renal blood flow, and activation of the sympathetic compensatory mechanisms. Signs and symptoms include shortness of breath and dyspnea (which may be exertional), orthopnea, paroxysmal nocturnal dyspnea, a chronic nonproductive cough, wheezing, fatigue, exercise intolerance, fluid retention and edema, cachexia, malnutrition, and cyanosis. Elevations in the jugular venous height may be measurable along with bibasilar crackles, and dullness to percussion may be evident. The cardiac exam may reveal a parasternal lift, an enlarged and sustained left ventricular impulse, a diminished first heart sound, S3 gallop, and an S4 with diastolic heart failure. Contributory murmurs may also be evident. Severe heart failure may present with diaphoresis and tachycardia. Pulmonary edema, the most dramatic symptom of acute heart failure, is characterized by patients gasping for air, tachycardia, moist and cool skin, cyanosis, confusion, and a productive cough with frothy (often blood-tinged) sputum. Chest pain, dyspnea, and tachypnea are the most frequent signs and symptoms of pulmonary embolism. Other manifestations may include tachycardia, pleurisy, low-grade fever, apprehension, and productive cough with blood-tinged sputum. Massive PE may manifest as sudden collapse, crushing substernal chest pain, shock, diaphoresis, hypotension, distended neck veins, and loss of consciousness. The presentation of myocardial infarction involves a prolonged, severe, and crushing sensation often described as tightness, squeezing, burning, pressing choking, aching, bursting, "gas," indigestion, or an ill-characterized discomfort. It is usually substernal, radiating to the left arm, neck, or jaw, but it may be perceived in other areas of the chest. Women may experience atypical ischemic-type chest discomfort; the elderly may experience shortness of breath. Associated anxiety, restlessness, feelings of impending doom, nausea, vomiting, skin pallor and coolness, diaphoresis, lightheadedness, and syncope may be present singly or in combination. Clinical manifestations of COPD are insidious and include fatigue, exercise intolerance, cough, sputum production, and progressive shortness of breath. Productive cough usually occurs in the morning. Emphysema-predominant COPD patients are referred to as "pink puffers" due to a lack of cyanosis, the presence of accessory muscle usage, and pursed-lip breathing. Patients are typically thin, and they demonstrate excessive accessory muscle use without adventitious lung sounds or peripheral edema. Loss of lung elasticity and hyperinflation leads to air trapping, reduced breath sounds, diffuse hyperresonance to percussion, and a barrel chest. The expiratory phase of respiration is prolonged, and expiratory wheezes and crackles can be heard. "Blue bloaters" represent the clinical picture of chronic bronchitis, which includes an overweight patient experiencing cough with mucopurulent sputum, mild dyspnea, peripheral edema, and lung rhonchi and wheezes. Signs and symptoms of pneumonia include a sudden onset of malaise, severe shaking chills, and fever. Coughing brings up watery sputum to rust-colored, purulent or blood-tinged sputum. Other findings include pleuritic chest pain and reduced breath sounds.

A 32-year-old G2P2 woman presents 1 month post-intrauterine contraceptive device placement for follow-up position check. The patient states that she can no longer feel the strings from the device. She was told to check occasionally to make sure it stayed positioned properly. Upon vaginal exam, you confirm that the strings are no longer visible in the cervical os. Question What is the most appropriate procedure to evaluate this patient? 1 CT scan 2 Laparoscopy 3 MRI 4 Ultrasound 5 X-ray

Answer: 4 US The fastest and best method to determine the location of an IUD is an ultrasound of the uterus. This can locate the IUD even if it has eroded through the lining of the uterus. This method is inexpensive, typically available in clinic, and does not expose the patient to radiation. If the IUD has perforated the uterus, then a laparoscopy may be necessary to remove the device. Other tests are not usually performed in this situation.

A 62-year-old woman presents to her physician with persistent reflux symptoms despite medical management. An upper endoscopy is performed, revealing the attached image. Biopsy findings reveal replacement of the esophageal squamous epithelium with columnar epithelium. Question What is the most serious complication of her disorder? 1 Peptic stricture 2 Esophageal adenocarcinoma 3 Esophageal rupture 4 Gastric adenocarcinoma 5 Gastric ulcer

Answer: esophageal Adenocarcinoma Esophageal adenocarcinoma is correct. The patient's biopsy reveals replacement of the esophageal squamous epithelium with columnar epithelium, which is consistent with Barrett's esophagus. The most serious complication of Barrett's esophagus is esophageal adenocarcinoma. Patients require routine endoscopic screening to rule out the development of esophageal cancer. Peptic stricture, esophageal rupture, gastric adenocarcinoma, and gastric ulcer are all incorrect. Esophageal adenocarcinoma is the most significant complication of Barrett's esophagus.

An 84-year-old man with poorly controlled hypertension is evaluated for severe chest pain that came on suddenly; the pain is accompanied by an episode of syncope. The chest pain is described as "ripping," and it radiates to the abdomen and back. There is no known history of trauma, injuries, or accidents. He denies any fever, chills, cough, dyspnea, palpitations, nausea, vomiting, or diarrhea. The physical exam is notable for tachycardia, a blood pressure of 188/120 mm Hg, and a high-pitched blowing decrescendo murmur at the right second interspace. What agent is the preferred initial treatment in the management of this patient? 1. Nitroprusside 2. Diltiazem 3. Morphine Sulfate 4. Enalapril 5. Labetalol

Answer: 5. Labetalol This patient is most likely experiencing an aortic dissection. Unless they are leaking or ruptured, medical management remains the treatment of choice for descending aortic dissections. Medical management consists of decreasing the blood pressure and the shearing forces of myocardial contractility in order to decrease the intimal tear and propagation of the dissection. Medical management with antihypertensive therapy, including beta blockers, is the treatment of choice for all stable chronic aortic dissections. The agent of choice for this patient consists of a negative inotropic medication, such as labetalol, as it has a dual effect of decreasing blood pressure and cardiac contractility. The target heart rate for patients with aortic dissection is 60-80 bpm. Nitroprusside causes peripheral vasodilation by direct action on venous and arteriolar smooth muscle, reducing peripheral resistance. Before initiating nitroprusside, a beta blocker should be used to offset the reflex tachycardia that occurs when nitroprusside is used alone. This physiologic response increases shear forces against the aortic wall. When beta blockers are contraindicated (second- or third-degree atrioventricular block), a calcium channel blocker such as diltiazem should be considered. Pain control is essential to quality patient care and may be achieved by morphine sulfate. It ensures patient comfort, promotes pulmonary toilet, and prevents exacerbations of tachycardia and hypertension, reducing the force of cardiac contraction and the rate of rise of the aortic pressure. It then retards the propagation of the dissection and delays rupture. ACE inhibitors may be added in conjunction to β-antagonists in the chronic drug therapy of aortic dissections.

A 58-year-old man with a history of COPD, hyperlipidemia, a 40 pack-year smoking history, and obesity is being evaluated at his primary care office for complaints of post-prandial regurgitation that is associated with an acidic sensation in his mouth and a chronic, nonproductive cough, all of which have been occurring over the past year. He has taken over the counter famotidine (Pepcid) without any relief. He denies any fever, chills, changes in weight, diaphoresis, chest pain, shortness of breath, sputum, palpitations, abdominal pain, or changes in bowel habits. His physical exam reveals an obese BMI, but it is otherwise unremarkable. His stool hemoccult is negative. Bloodwork reveals an iron-deficiency anemia; a chest radiogram was without pulmonary disease. An upper barium esophagram noted an outpouching of barium at the lower end of the esophagus and a wide hiatus through which gastric folds are visible above the diaphragm. H. pylori antibody testing and urea breath tests were negative. What is the most appropriate therapy for this patient at this time? 1 Esomeprazole (Nexium) 2 Ranitidine (Zantac) 3 Metoclopramide 4 Nitroglycerin 5 Metronidazole (Flagyl), tetracycline, and bismuth subcitrate

Answer: 1 Esomeprazole Esomeprazole (Nexium) is the correct response. This presentation is most consistent with a sliding hiatal hernia. Barium esophagograms or swallows are helpful for identifying structural abnormalities of the esophagus and esophageal hiatus, which include esophageal rings, strictures and ulcers, and hiatal hernias. Large hiatal hernias may cause iron deficiency anemia regardless of whether Cameron ulcers are present. This anemia responds well to PPI therapy, with surgery offering no clear advantage over medical therapy. PPIs are also superior to H2 receptor antagonists for the resolution of associated GERD symptoms at 4 weeks and the healing of esophagitis at 8 weeks. This patient remains symptomatic despite initial H2-receptor antagonist therapy; ranitidine would not be appropriate at this time. Nitroglycerin is not indicated, as this patient is not suffering from angina pectoris. Prokinetic agents are somewhat effective, but only in patients with mild symptoms; other patients usually require additional acid-suppressing medications, such as PPIs. The usual regimen in adults is metoclopramide. Long-term use of prokinetic agents may have serious—even potentially fatal—complications and should be discouraged. Antibiotics with a proton pump inhibitor for eradication of H. pylori would be suitable with evidence of H. pylori infection, but there is none in this patient.

49-year-old Caucasian man who is well known to your practice presents due to his history of hypertriglyceridemia. He has come in to see you today for evaluation of his recent cholesterol lab values. He has a significant family history of cardiovascular disease; his mother had a heart attack at the age of 57, and his father had open heart surgery at the age of 60. The patient has had low HDL levels in several past cholesterol screening tests. At the last office visit, lifestyle modifications were implemented by the patient. He has been extremely conscientious about his diet; for the past year, he has also been participating in physical activity 6 days a week. In addition, he has significantly limited his alcohol intake. Current fasting lab values for the patient are as follows: total cholesterol of 235 mg/dL, triglycerides of 350 mg/dL, HDL of 35, and an LDL of 175 mg/dL. You decide to initiate pharmacologic therapy in order to treat the patient's cholesterol and triglyceride levels. Question What medication should you prescribe? 1 Rosuvastatin 2 Ezetimibe 3 Colestipol 4 Simvastatin 5 Cholestyramine

Answer: 1 Rosuvastatin Most patients with high cholesterol levels present with no specific symptoms or signs, although this is a significant contributor to the number one killer of adults in the United States: cardiovascular disease. There are a multitude of modifiable as well as non-modifiable risk factors present that contribute to overall cholesterol levels. If a patient has any abnormal cholesterol levels, the initial component of any treatment is appropriate patient education with lifestyle modifications, which many times include limiting consumption of alcohol and cholesterol-containing foods, smoking cessation, and increasing physical activity. If a valiant effort to address modifiable risk factors fails to bring cholesterol levels to normal ranges, pharmaceutical intervention should be initiated. The patient described in the scenario above would most benefit from the initiation of rosuvastatin. This medication is a high-intensity statin, which will aid in reduction of LDL. This particular statin also has a more significant effect on triglycerides as compared to other statins. Simvastatin is a low-intensity statin. It will address LDL to some degree, but not as effectively as rosuvastatin. Simvastatin also does not aid in the lowering of triglycerides nearly as well as rosuvastatin. Ezetimibe is a cholesterol absorption inhibitor. It has a modest effect on LDL levels (one study showed a 17% reduction) but does not aid appreciably in lowering triglycerides, which is one of this patient's primary abnormalities. Cholestyramine and colestipol are bile acid sequestrants, which may help lower the patient's triglycerides, but will not address his elevated LDL, and therefore are not the best choices given his strong family history of CVD.

An 86-year-old man presents with a history of a dislocated right hip. For the past 3 weeks, he has noticed pain where his hip dislocated when putting weight onto it. He reports a history of alcohol abuse and use of steroids for his rheumatoid arthritis. Upon physical exam, there is limited range of motion in the right hip but he is afebrile. An X-ray of the right hip shows sclerosis of the bone; however, a synovial fluid analysis is normal. Question What is the most likely diagnosis? 1 Avascular necrosis 2 Legg-Calvé-Perthes Disease 3 Osteochondritis dissecans 4 Septic arthritis 5 Sesamoiditis

Answer: 1 avascular necrosis The clinical picture is suggestive of avascular necrosis. Avascular necrosis commonly happens after a traumatic injury such as a dislocation. Additionally, alcohol use and steroid use are noted, which can be risk factors for avascular necrosis. Lastly, the findings demonstrate sclerosis of the bone on the X-ray. Legg-Calvé-Perthes Disease is not the correct answer, as Legg-Calvé-Perthes Disease occurs commonly in the age range of 4-9. Patients with Legg-Calvé-Perthes Disease have risk factors, such as maternal smoking during pregnancy and low birth weight. Osteochondritis dissecans is not correct because osteochondritis dissecans is a lesion of subchondral bone that causes a separation and instability overlying articular cartilage. Additionally, osteochondritis dissecans more commonly occurs in adolescents. Septic arthritis is not the correct answer because septic arthritis most commonly presents with a fever, which this patient does not have. Additionally, a synovial fluid analysis of 145 WBC/mm3 is in a normal range and therefore rules out septic arthritis. Sesamoiditis is not correct because sesamoiditis affects the sesamoid bones under the first metatarsophalangeal joint.

A 6-year-old girl presents with hematuria. Review of the child's prior medical records reveals approximately 20 visits to 8 different emergency departments and clinics over the past year for the same complaint. Thorough urologic evaluations have been negative, except for the occasional and inconsistent finding of elevated red blood cells on urinalysis without casts. The child is admitted to the general pediatric ward. Physical examination is negative except for mild erythema of the urethral meatus. Initial laboratory evaluation, including a serum BUN of 8 mg/dL, serum creatinine of 1.0 mg/dL, serum total protein of 7.1 mg/dL, and serum albumin of 4.3 mg/dL, is normal. Urinalysis performed on a sample brought to hospital by the girl's mother is positive for blood. This sample is grossly bloody, but samples subsequently obtained in hospital under nursing supervision are negative. Renal ultrasonography is normal. The child's mother appears quite cheerful and is rather conversant with medical terms concerning renal disease such as glomerulonephritis and gross hematuria. She is happy to suggest additional diagnostic tests. Question What is the most appropriate next step in the evaluation of this child? 1 Consultation with child protective services 2 Hematology consultation to rule out coagulopathy 3 Intravenous pyelography 4 Surreptitious video monitoring of the child's room 5 Urology consultation

Answer: 1 consultation with child protective services Multiple negative evaluations, inconsistent positive findings, the use of multiple healthcare facilities, and inappropriate maternal affect should suggest a condition fabricated by a caregiver. Once called "Munchhausen syndrome by proxy," this scenario can be taken to dangerous lengths, sometimes causing multiple invasive and unnecessary surgeries and diagnostic procedures. It is considered a form of child abuse, so the situation warrants the involvement of child protective services. The absence of conclusive evidence of hematuria, as well as the lack of historical or physical findings of bleeding elsewhere, renders a coagulopathy unlikely. The lack of conclusive evidence of hematuria and evidence of normal renal function do not support performing intravenous pyelography. While video monitoring may gather evidence of the mother manipulating urine specimens or of injuring the child, the physician should not be cast in the primary role of detective in this situation. Such efforts should be coordinated by Child Protective Services. Again, the absence of any real evidence of renal or genitourinary pathology argues against a urology consultation as the next step.

A 62-year-old woman with past medical history of hyperlipidemia presents due to shortness of breath. She is a nonsmoker, but she drinks a pint of vodka daily. Chest X-ray reveals severe cardiomegaly. She is ultimately diagnosed with cardiomyopathy. Question What type of cardiomyopathy does the patient most likely have? 1 Dilated cardiomyopathy 2 Hypertrophic cardiomyopathy 3 Restrictive cardiomyopathy 4 Takotsubo cardiomyopathy 5 Postpartum cardiomyopathy

Answer: 1 dilated cardiomyopathy The above patient is most likely suffering from dilated cardiomyopathy. Dilated cardiomyopathy occurs when the left ventricle dilates and is unable to pump blood effectively. Patients may develop symptoms of congestive heart failure, with dyspnea as the most common presenting complaint. Chest X-ray will reveal cardiomegaly, due to the dilatation of the left ventricle. Alcohol abuse is a risk factor for the development of dilated cardiomyopathy. Patients with dilated cardiomyopathy are encouraged to abstain from alcohol use. Hypertrophic cardiomyopathy is less likely, given the patient's age. Patients with hypertrophic cardiomyopathy typically present in early adulthood and have a familial history of hypertrophic cardiomyopathy. Chest X-ray may reveal cardiomegaly, but it is typically less marked than what would be seen in dilated cardiomyopathy. Restrictive cardiomyopathy is less likely given the patient's history of alcohol abuse. Restrictive cardiomyopathy is relatively rare and is typically caused by amyloidosis. Chest X-ray may reveal cardiomegaly, but the patient's past medical and social history make dilated cardiomyopathy more likely. Takotsubo cardiomyopathy is unlikely given the patient's chest X-ray findings. Patients with Takotsubo cardiomyopathy typically do not have cardiomegaly on chest X-ray. Postpartum cardiomyopathy is an incorrect answer; the patient is not of childbearing age.

A 43-year-old man without any significant PMHx presents with acute onset of a productive cough, shortness of breath, pleuritic chest pain, and fever. His sputum is described as "thick, brown-colored, and mucoid" but without blood. He also notes associated fatigue and night sweats. He denies chills, changes in weight, a history of travel, sick or confined contacts, exposure to animals, cigarette smoking, otalgia, sore throat, swollen glands, abdominal pain, diarrhea, rashes, myalgias, and arthralgias. His physical exam is remarkable for fever, tachycardia, and tachypnea. The lung exam is noteworthy for right lower lung field increased tactile fremitus, dullness to percussion, inspiratory crackles, and bronchial breath sounds. Question What is the most likely etiologic agent responsible for this patient's presentation? 1 Streptococcus pneumoniae 2 Histoplasma capsulatum 3 Chlamydia psittaci 4 Pseudomonas aeruginosa 5 Mycobacterium tuberculosis

Answer: 1 strep pneumo This patient is demonstrating signs and symptoms representative of bacterial pneumonia. Streptococcus pneumoniae is the most common cause of community-acquired pneumonia and classically causes rust-colored sputum. Travel to the Ohio or St. Lawrence River valley or contact with bird droppings and bat areas should raise suspicion of Histoplasma capsulatum pneumonia. The presentation of acute histoplasmosis consists of marked prostration, fever, but a relative scarcity of pulmonary issues despite the potential for significant chest X-ray findings. Exposure to birds (parrots, parakeets, pigeons, chickens, ducks) 7-15 days prior to the development of rapid onset fever, chills, myalgia, dry cough, and headache is consistent with psittacosis. Gram-negative pneumonias (e.g., Pseudomonas aeruginosa) occur most often in individuals who are debilitated, immunocompromised, or recently hospitalized. Individuals living in long-term care facilities where other residents are intubated are also at risk for these infections. Pseudomonas may cause greenish sputum. Tuberculosis is a caseating granulomatous infection characterized by fever, followed by night sweats, malaise, fatigue, and weight loss. Productive cough, hemoptysis, dyspnea, and pleuritic chest pain develop as the infection spreads within the lungs. Homeless patients, immigrants, those with HIV, those who use drugs, alcoholic patients, and elderly patients are at the greatest risk.

A 4-year-old girl presents with her mother to discuss treatment of her atopic dermatitis. She was diagnosed as an infant, but her case appears to be getting worse despite frequent lubrication with thick emollient creams and medium potency topical corticosteroid use. Mother states that the patient is itching a lot more, especially during the night. Large, single patches of erythematous scaly excoriations, measuring about 3 cm by 4 cm, are present in the flexor surfaces of both the right and left elbows. Question What second line treatment is a reasonable choice to replace her current regimen now that the disease has become more severe? 1 High potency topical corticosteroids 2 Topical antihistamines 3 Oral antihistamines 4 Systemic anti-Staphylococcal antibiotics 5 Ketoconazole cream

Answer: 1 high potency topical corticosteroids High potency topical corticosteroids' is the correct answer. Since a medium potency corticosteroid topical has been attempted, it would be most efficient to now try a step stronger in regards to topical steroids. 'Oral antihistamines' is not the correct answer. Oral antihistamines are given to patients with atopic dermatitis if they have pruritus that requires treatment, but do not actually treat the condition itself. Medications such as hydroxyzine can be given at bedtime as needed. 'Systemic anti-Staphylococcal antibiotics' is not the correct answer. It is possible for patients with atopic dermatitis to become superinfected with bacteria, commonly Staphylococcus aureus. In addition to treatment for the atopic dermatitis, patients can be placed on systemic antibiotics for 10-14 days. 'Ketoconazole cream' is not the correct answer. Ketoconazole cream is an antifungal cream used to treat various other skin disorders, but not atopic dermatitis. Seborrheic dermatitis typically has yeast on the skin surface and can be treated with ketoconazole. However, atopic dermatitis does not have a yeast or fungal component, and therefore is not treated with ketoconazole.

A 23-year-old man presents with a 2-day history of watery nasal discharge, cough, sore throat, and hoarseness of voice. On examination, pulse is 80/min. BP is 130/84 mm Hg, and temperature is 98.8°F. Nasopharyngolaryngoscopy shows laryngeal edema and erythema. The rest of the examination is normal. Question What should be the treatment for this patient? 1 Humidification and voice rest 2 Antibiotics and non-steroidal anti-inflammatory drugs (NSAIDs) 3 Antiviral drugs and non-steroidal anti-inflammatory drugs (NSAIDs) 4 Antibiotics alone 5 Antiviral drugs alone

Answer: 1 humidification and voice rest The likely diagnosis is acute viral laryngitis, which is treated with humidification and voice rest. Antibiotics are not recommended except when group A Streptococcus is cultured, in which case penicillin is the drug of choice (unless the strain is known to be penicillin-resistant). Antivirals against influenza A and respiratory syncytial virus are used only in cases involving immunocompromised patients.

A 35-year-old man with no significant past medical history presents with multiple skin lesions in various parts of his body. These evolved gradually over the past year. He describes them as small, painless, non-pruritic, and red-yellow, as well as flesh-toned. They are distributed on his buttocks, posterior ankles, and on his knees. There is no discharge to these lesions. He denied any allergies, trauma, history of surgeries, or instrumentation. He further denied recent fever, chills, travel, insect bites, sick contacts, chest pain, cough, abdominal pain, nausea, vomiting, diarrhea, edema, or other rashes. The physical examination was notable for an overweight man with multiple red-yellow papules on the buttocks. Additionally, there were several flesh-toned, firm, nodular lesions distributed over the bilateral Achilles and patellar tendons. All skin lesions were nontender and without discharge. The remainder of the physical exam was unremarkable. Question What is the most likely diagnosis? 1 Hyperlipidemia 2 Squamous cell carcinoma 3 Folliculitis 4 Psoriasis 5 Nummular eczema

Answer: 1 hyperlipidemia This patient is presenting with hyperlipidemia. Extremely high levels of chylomicrons or VLDL particles (triglyceride level above 1000 mg/dL) result in the formation of eruptive xanthomas. These lesions are described as red-yellow papules, which are commonly found on the buttocks. High LDL concentrations result in tendinous xanthomas on certain tendons, such as the Achilles, patellar, and those on the back of the hand. The lesions associated with squamous cell carcinoma occur primarily in chronic sun-exposed areas and appear as small red, conical, hard nodules that occasionally ulcerate without healing. Folliculitis typically presents as pustules of hair follicles, with symptoms ranging from slight burning and tenderness to intense itching. Psoriatic lesions may be pruritic and are most commonly found on the scalp, elbows, knees, palms, and soles. They are red, sharply defined plaques covered with silvery scales. Pitting and onycholysis are common nail findings. Nummular eczema is a chronic, pruritic inflammatory dermatitis occurring as coin-shaped plaques composed of grouped small papules and vesicles on an erythematous base, common on the lower legs. These are seen in atopic individuals.

A 35-year-old Hispanic man presents to your office but is too embarrassed to tell the nurse his chief complaint. You enter the room, and he admits to severe, intense itching around his anus that has been worsening the last several weeks. He further states that he has noticed increasingly severe and tearing pain in the anal area with each bowel movement. He would rank this pain as a 10/10 on a pain scale. This intense pain makes him not want to have any bowel movements. He admits to only one episode of a small amount of bright red blood on the toilet paper as well as on the stool itself. The patient denies fever, diarrhea, or ever being diagnosed with inflammatory bowel disease. Question Considering the findings in the patient above, what is best choice of diagnostic study to assist in confirmation of diagnosis? 1 Physical examination 2 Digital rectal exam 3 Anoscopy 4 Sigmoidoscopy 5 Colonoscopy

Answer: 1 physical examination This patient is most likely suffering from an anal fissure. Anal fissures most often affect infants and middle-aged individuals. The majority of fissures are considered primary and caused by local trauma, such as passage of hard stool, prolonged diarrhea, vaginal delivery, or anal sex. Presentation of anal fissures is a tearing pain accompanying bowel movements as well as bright red rectal bleeding that is limited to a small amount noted on the toilet paper or surface of the stool. The patient described all of these components. Patients will also complain of perianal pruritus or irritation, which he also admits to experiencing. The best diagnostic approach to confirming an anal fissure is usually based on the history of pain with defecation and the physical examination finding of a superficial tear noted in the anoderm; this type of tear, no matter if located posteriorly or laterally, is considered a pathognomonic feature of an acute anal fissure. The examiner should carefully spread the buttocks apart and examine the area gently. Digital rectal examination, or anoscopy, is not recommended for diagnosis of this condition; patients will not be able to tolerate them due to the severe pain they will be experiencing from the anal fissure. Sigmoidoscopy or colonoscopy are both inappropriate diagnostic tools in this patient case scenario. These studies should only be considered if there is presence of otherwise unexplainable rectal or gastrointestinal bleeding. Either a sigmoidoscopy or a colonoscopy can assist in evaluating the corresponding source of bleeding.

You are following up on laboratory results for your supervising physician while she is out of town. A bone marrow biopsy result for a 62-year-old man is noted: Infiltration with small lymphocytes, B-cell. Coexpression of CD19, CD20 and CD5. On further chart review, you see that this patient presented to your clinic for review of abnormal health fair screening tests for his employment. The patient reported that he felt well in general, but he added that he could not exercise as vigorously as he could in the past; he attributed that to his aging. The table shows his complete blood count. Test Result Normal range Units WBC 98.4 3.6-9.0 K/μL RBC 4.78 4.18-5.22 M/ μL Hgb 15.4 12.9-15.5 g/dL Htct 45.2 34.6-50.1 % MCV 94.6 80.0-100.0 fL MCH 32.2 27.0-34.0 pg MCHC 34.1 30.0-37.0 g/dL RDW 11.7 11.0-17.0 % Platelets 392 140-440 K/μL MPV 9.9 6.5-12.0 fL WBC differential Neutrophils 5.70% 40-75% Lymphocytes 91% 20-50% Monocytes 2% 0-10% Eosinophils 1.20% 0-7% Basophils 0.10% 0-3% Question Once this patient has been referred for treatment and achieves disease remission, he returns to his primary care clinic for health maintenance. What complication specifically related to his condition should the primary care physician monitor for? 1 Secondary malignancy 2 Decreased visual acuity 3 Hypercoagulability 4 Osteoporosis 5 Valvular heart disease

Answer: 1 secondary malignancy This patient's bone marrow result and presentation support a diagnosis of chronic lymphocytic leukemia (CLL), the most prevalent type of adult leukemia. CLL typically affects older individuals and can remain at a mild stage with minimal symptoms for years. Approximately 10-20% of patients with CLL are found to have a second malignant tumor, such as skin, colorectal, or lung cancer. Patients diagnosed with CLL should be educated on their risk for another cancer, and appropriate screening should take place. Decreased visual acuity, hypercoagulability, osteoporosis, and valvular heart disease are not specifically related to CLL; their treatments are not used for CLL.

A 15-year-old boy presents for follow-up of previously diagnosed Asperger syndrome, now autism spectrum disorder. The patient is accompanied by his mother. He is considered high functioning. He attends public school, goes to daily cognitive/behavioral sessions, and attends weekly counseling sessions. The mother states she has noted a significant increase in the patient's anxiety symptoms that have become detrimentally disruptive to daily activities. Question What would be your initial choice of pharmacologic treatment to help decrease the effects of this patient's anxiety? 1 Sertraline 2 Duloxetine 3 Atomoxetine 4 Clonidine 5 Methylphenidate

Answer: 1 sertraline Adolescents with autism spectrum disorder will have varying degrees of impairment in their social and behavioral function, so treatment must be individualized based on the patient's age and their needs. Family education, behavioral and educational interventions, and counseling have a significant place in this treatment plan. Pharmacotherapy may be considered, but should be used as adjunctive therapy. Children and adolescents with AS are treated via pharmacotherapy mainly to assist in control of their psychiatric symptoms; these patients are much more sensitive to medication effects. It is suggested to initially start the patient on a selective serotonin reuptake inhibitor (SSRI) (e.g., sertraline or fluoxetine) for anxiety symptoms. Duloxetine is a serotonin-norepinephrine reuptake inhibitor (SNRI). SNRIs are utilized in a multitude of patients, including those who are diagnosed as having major depressive disorder, for treatment of musculoskeletal pain in osteoarthritis, peripheral neuropathy, and even fibromyalgia. Although duloxetine is seen as a pharmacologic option for patients with generalized anxiety disorder, there is no hard recommendation for usage in AS patients to treat their anxiety symptoms. Patients who express symptoms consistent with hyperactivity and inattention would be treated with methylphenidate, atomoxetine, or clonidine, but these symptoms are not seen in this patient. When beginning any pharmacotherapy for these patients, having a "start low and go slow" regimen is strongly recommended, with consistent follow-up visits for evaluation of alleviation of the symptoms.

A 61-year-old woman presents a 1-week history of intermittent episodes of feeling like she was spinning. She states the episodes are brief, but they occur 2-3 times per day. It is worse when she turns to her right side while lying in bed. Even when she is not dizzy, she feels off balance. She denies tinnitus, decreased hearing, fever, syncope, nausea, vomiting, diplopia, or any other related symptoms. During the Dix-Hallpike maneuver, the patient exhibits nystagmus, with her eyes beating upward and torsionally when the right ear is turned downward. The nystagmus diminished with each time the maneuver was performed. Based on the above description, what is the most likely diagnosis? 1 Benign paroxysmal positional vertigo 2 Labyrinthitis 3 Meniere's disease 4 Vestibular schwannoma 5 Brainstem infarction

Answer: 1. BPPV Benign paroxysmal positional vertigo (BPPV) episodes are brief in nature and occur with changes in position. People often feel off balance even when an episode is not occurring. BPPV does not typically cause hearing loss. The Dix-Hallpike maneuver elicits an episode of vertigo, with nystagmus being noted during the exam. The nystagmus diminishes with each maneuver due to fatiguability. Labyrinthitis is usually caused by an infection, which the patient did not have. Labyrinthitis often includes hearing loss and or tinnitus, with episodes often lasting days or weeks. Vertigo with Meniere's disease usually lasts from 20 minutes to 24 hours. Sensorineural hearing loss and tinnitus are also common features. Typical onset is usually age 20-40. Most people with vestibular schwannoma this have hearing loss and or tinnitus. True spinning vertigo is uncommon with this disorder. People do have unsteadiness with walking. Some other symptoms can include paresthesia, hypesthesia, facial paresis, and taste disturbances. Symptoms are slow onset. Brainstem infarction patients present with sudden onset of symptoms, and the symptoms persist for days to weeks. Nystagmus has central characteristics, as opposed to the above patient, who showed peripheral characteristics. This would also have associated neurological signs and symptoms.

A 69-year-old man presents with dyspnea on exertion that has been slowly progressive over the course of the last year. He notes impairment in climbing stairs and walking short distances. His review of systems is positive for fatigue, palpitations, intermittent retrosternal chest pain, swelling of his lower extremities, dizziness, and "feeling faint." His associated symptoms are also known to occur upon exertion. He denies any fever, chills, weight changes, cough, abdominal pain, early satiety, nausea, vomiting, diarrhea, changes in his urine color or odor, flank pain, hematuria, or dysuria. He denies any cigarette, alcohol, or drug use. His cardiac exam is remarkable for an increased pulmonic component of the second heart sound (P2), wide, inspiratory splitting of S2 over the cardiac apex, right-sided S3 and S4 gallops, a left parasternal lift, a loud diastolic murmur that increases with inspiration and diminishes with the Valsalva maneuver, prominent "A" waves in jugular venous pulsations, and increased JVD. He has an enlarged liver with hepatojugular reflux, peripheral edema, and ascites. A bedside EKG analysis revealed peaked P waves, rightward axis deviation, and prominent R waves in the early V leads. Question What is the most likely diagnosis? 1 Myocardial infarction 2 Cor pulmonale 3 Primary biliary cirrhosis 4 Left ventricular heart failure 5 Pulmonary embolism

Answer: 2 Cor pulmonale The most appropriate diagnosis in this scenario is cor pulmonale, often referred to as pulmonary heart disease. It occurs due to right ventricle structure and/or function alteration due to chronic lung disease, and it's usually triggered by pulmonary hypertension. Symptoms that present are due to the underlying pulmonary pathology. Dyspnea is the most common symptom, as well as abdominal pain and ascites. Signs found on physical assessment include tachypnea, elevated jugular venous pressure, hepatomegaly, and lower extremity edema. Evidence of prominent V waves in the jugular venous pulse may be found, as well as an RV heave palpable along the left sternal border or even in the epigastrium. Cyanosis is a late finding in the course of cor pulmonale. Diagnostically, ECG findings will display P pulmonale, right axis deviation, and right ventricle hypertrophy. Myocardial infarctions occur at rest and most commonly in the early morning. The pain is similar to angina in location and radiation, but it may be more severe and builds up rapidly or in waves to maximum intensity over a few minutes or longer. Associated symptoms include diaphoresis, weakness, apprehension, and a feeling of impending doom; patients may move about, seeking a position of comfort, preferring not to lie quietly. Light-headedness, syncope, dyspnea, orthopnea, cough, wheezing, nausea and vomiting, or abdominal bloating may occur. Physical exam findings may include fever, anxiousness, diaphoresis, bradycardia or tachycardia, low cardiac output, or arrhythmia. There may be hypertension (in hypertensive patients) or low in patients with shock. Respiratory distress, jugular venous distention, a Kussmaul sign, soft heart sounds, and atrial gallops (S4) or ventricular gallops (S3) usually indicate heart failure. Primary biliary cirrhosis is most common in middle-aged females and is characterized by fatigue, pruritus, hepatosplenomegaly, xanthomatous lesions on the skin, eyelids, and tendons, jaundice, and steatorrhea. Signs of portal hypertension are late findings. Other findings include orthostatic hypotension and cognitive dysfunction. Left-sided or forward failure may account for many of the clinical manifestations of heart failure, such as mental confusion from decreased cerebral perfusion, fatigue and weakness from decreased skeletal muscle perfusion, and sodium and water retention with secondary venous congestion from decreased renal perfusion. Isolated left-side heart failure is associated with dyspnea, fatigue, weakness, cough, paroxysmal nocturnal dyspnea, and orthopnea in the absence of peripheral edema, jugular venous distention (JVD), or hepatojugular reflux. Chest pain, dyspnea, and tachypnea are the most frequent signs and symptoms of pulmonary embolism. Other manifestations may include tachycardia, pleurisy, low-grade fever, apprehension, and productive cough with blood-tinged sputum. Massive PE may manifest as sudden collapse, crushing substernal chest pain, shock, diaphoresis, hypotension, distended neck veins, and loss of consciousness.

22-year-old man presents to his physician with swollen lymph nodes in the right axilla. He notes that he develops pain in the area after drinking alcohol. He has been feeling fatigued for the last few weeks and has lost weight without trying. Examination confirms lymphadenopathy in the right axilla. Biopsy of the region shows the presence of Reed Sternberg cells. Question What is the treatment of choice at this time? 1. Lenalidomide and prednisone 2 Doxorubicin, bleomycin, vinblastine, and dacarbazine 3 Radiation therapy 4 Stem cell transplant 5 Observation

Answer: 2 Doxorubicin, bleomycin, vinblastine, and dacarbazine The combination of doxorubicin, bleomycin, vinblastine, and dacarbazine (ABVD) is the treatment of choice in Hodgkin's lymphoma. The patient's presentation of lymphadenopathy with the presence of Reed Sternberg cells on biopsy is classic for Hodgkin's lymphoma. The combination of lenalidomide and prednisone is typically used in the treatment of multiple myeloma. Radiation therapy may be used in the treatment of Hodgkin's lymphoma, but it is usually used in combination with chemotherapy. Stem cell transplant may be used in the treatment of Hodgkin's lymphoma, but it is not the first line. Observation is incorrect. Hodgkin's lymphoma requires treatment.

A 24-year-old African American G2P1 presents to her obstetrician at 34 weeks gestation with weight gain, fatigue, diffuse "swelling," and headache. On physical examination, you note periorbital edema and 3+ pitting edema in the lower extremities peripherally. Physical examination also demonstrates tenderness to palpation in the right upper quadrant. The patient has increased reflexes bilaterally. Vital signs are as follows: Temperature is 99.2°F. Pulse is 89 beats/minute. Respirations are 18. Blood pressure is 174/99 mm Hg. Urinalysis reveals no erythrocytes per high power field and no casts, but there is a large amount of protein in the urine. Laboratory values are as follows: Test Result Normal Range Hemoglobin 9.2 g/dL 13.0-16.0 g/dL Hematocrit 27% 37-47% Platelets 38 K/mm3 130-400 K/mm3 WBC 12.7 K/μL 3.8-10.8 K/μL AST 595 IU/L 7-40 IU/L ALT 585 IU/L 0-40 IU/L Alkaline phos 295 U/L 70-230 U/L Question What is the most likely diagnosis? 1 Disseminated intravascular coagulation (DIC) 2 Hemolysis, elevated liver tests, and low platelets syndrome (HELLP) 3 Idiopathic thrombocytopenic purpura (ITP) 4 Thrombotic thrombocytopenic purpura (TTP) 5 Normal laboratory values of pregnancy

Answer: 2 Hemolysis, elevated liver tests, and low platelets syndrome (HELLP) These laboratory findings are not normal, and the clinical history and low platelets are most consistent with HELLP syndrome. HELLP Syndrome is most often associated with third-trimester pre-eclampsia, HTN, and proteinuria. The acronym HELLP stands for Hemolysis, Elevated Liver Tests, and Low Platelets. While most patients with pre-eclampsia can be managed expectantly, patients with HELLP need to be induced for delivery (based on the maturity of the fetus), given the high risks associated with this condition. Patients with HELLP syndrome also need to be given magnesium sulfate to prevent seizure activity. Disseminated intravascular coagulation (DIC) can occur as a consequence of HELLP syndrome, placental abruption, sepsis, amniotic fluid embolism, or intrauterine fetal demise. Patients with DIC have clinical manifestations, including epistaxis, hematuria, purpura, and bleeding from puncture sites or incision sites. Fibrinolysis is an important component of DIC, and laboratory values show elevated levels of D-dimer and fibrin degradation products (FDPs). Platelet counts are usually low; clotting times (aPTT and PT) are elevated; antithrombin levels and individual clotting factors (V and VII) may be diminished. Given the lack of associated risk factors, DIC would be an unusual diagnosis in this patient. Idiopathic thrombocytopenic purpura (ITP) is diagnosis of exclusion and reflects a disease of increased peripheral platelet destruction, as most patients have antibodies to specific platelet membrane glycoproteins. Patients usually have physical findings of thrombocytopenia, including purpura, increased bruising tendencies, and epistaxis. In this patient with known ITP, high-dose parenteral glucocorticoids and IV immunoglobulin (IVIG), with or without platelet transfusions, are appropriate treatment. Splenectomy is usually reserved for patients when medical therapy fails. Thrombotic thrombocytopenic purpura (TTP) is the multi-system condition characterized by microangiopathic hemolysis and platelet aggregation with hyaline thrombi that forms unrelated to coagulation system activity. In TTP, platelet microthrombi predominate, forming in systemic arterioles and capillaries, leading to partial occlusion of these small vessels. Organ ischemia, thrombocytopenia, and erythrocyte fragmentation ensue. Additionally, most patients have altered mental status, including confusion, generalized headaches, focal deficits, seizures, visual disturbances, and/or coma. Renal insufficiency and gross hematuria may also be seen.

A 35-year-old man presents with a 2-week history of constant burning and throbbing pain in his left hand. The pain also seems to affect his distant forearm. Contact with normal clothing and bed sheets worsen his hand pain. He also reports his hand is swollen. He denies trauma to his hand, but he does report an uncomplicated left wrist fracture 3 months ago that resulted from a sports accident. By the time of cast removal at 7 weeks post-injury, the patient denied any pain or edema in the affected limb. He was instructed to return to normal activities, as tolerated. He was initially treated with ibuprofen and opioid pain pills. He discontinued all pain medicines within 2 weeks of the fracture. He now describes his hand pain as an 8 out of 10, on a 1-10 pain scale. On physical exam, the left hand and forearm are noted to have some localized edema, warmer temperature, and increased hair growth, compared to his right hand and forearm. No rash or skin lesions are noted. With even light palpation of the affected region, the patient cries out in pain. Range of motion is decreased, and reflexes are increased in the left upper extremity (in comparison to the right). The rest of his exam is normal. X-ray of the left wrist and hand are normal, with good fracture resolution. What actions might have best prevented this patient's complication? 1 Initiation of broad-spectrum antibiotics at time of fracture 2 Initiation of clopidogrel (Plavix) immediately after treatment of fracture 3 Initiation of vitamin C following fracture 4 More aggressive pain management following fracture 5 Prolonged immobilization with hard cast

Answer: 2 Initiation of vitamin C following fracture This patient most likely has complex regional pain syndrome (CRPS). CRPS is indeed a complex syndrome often occurring in a limb; patients exhibit vasomotor and neurologic symptoms out of proportion with objective findings. It often occurs following some type of event, such as a surgery, fracture, stroke, or myocardial infarction. When fracture is the inciting event, initiation of vitamin C following the fracture is associated with lower risk of CRPS.The reason is unknown. CRPS is classified into three stages, and symptoms can vary depending on the stage. It is characterized by persistent burning and/or throbbing pain in an extremity. Swelling, redness, and increased hair/nail growth occur in stage I; it may progress to cool, pale skin and even osteoporosis in the latter two stages. Allodynia, the phenomenon in which a normally painless stimulus produces significant pain response, is a common finding. No single exam finding or test is diagnostic of CRPS, which is a clinical diagnosis. This patient's painful erythematous hand could be mistaken for cellulitis, but cellulitis is not typically associated with closed wrist fracture. Initiation of broad-spectrum antibiotics at the time of an uncomplicated fracture is not recommended. One could consider a venous thrombosis as the cause of this patient's symptoms (although unusual to present in the hand and not associated with wrist fractures). Initiation of clopidogrel (Plavix) immediately after treatment of this fracture would not be recommended, and no current data suggest anticoagulants could have prevented this patient's CRPS. In contrast, anticoagulation should be considered following hip fractures, especially in vulnerable geriatric populations. No current data support that more aggressive pain management following fracture reduces the likelihood of CRPS. In fact, this patient reports that his pain resolved appropriately quite soon after the fracture. Prolonged immobilization with a hard cast would have likely contributed to further problems; there would have been more muscle atrophy, and there would have been no reduction of CRPS incidence following the fracture.

68-year-old man with a past medical history of type 2 diabetes mellitus, hypothyroidism, and hyperlipidemia presents with a constant moderate to severe "squeezing, pressure, and tight" left-sided chest pain for 1 hour. Additionally, he admits to shortness of breath, nausea, productive cough with a frothy sputum, and profound diaphoresis. He notes that he has had a 1-week history of similar, recurrent chest pain of about 10 minutes' duration that has been occurring following exposure to the cold weather and following consumption of a meal. He denies chills, abdominal pain, diarrhea, cough, and pleurisy. His physical exam reveals tachycardia, hypertension, cyanosis, cool and moist skin, diaphoresis, an S3 gallop, and evidence of painful respiratory distress. His lung exam is noteworthy for diffuse crackles. Bedside electrocardiogram demonstrates sinus tachycardia, ST segment elevations, and occasional premature ventricular contractions. Question What is the most appropriate treatment for the pain, hypertension, and suspected pulmonary edema in this patient? 1 Clopidogrel 2 Nitroglycerin 3 Metoprolol 4 Lisinopril 5 Nifedipine

Answer: 2 Nitroglycerin This patient's most likely diagnosis is acute ST-segment elevation myocardial infarction with pulmonary edema. Control of pain in STEMI is accomplished through a combination of nitrates, analgesics (morphine), oxygen, and β-adrenergic blocking agents. Nitroglycerin is indicated for the treatment of ongoing ischemic pain, control of hypertension, or management of pulmonary congestion. Due to its vasodilating effect, it is able to relieve coronary pain. It decreases venous return (reducing preload) and arterial blood pressure (reducing afterload), thereby reducing oxygen consumption. It may also limit infarction size. Clopidogrel reduces platelet aggregation by inhibiting the ADP pathway in platelets. Metoprolol, a β-adrenergic blocking drug, decreases myocardial oxygen demand by reducing heart rate, cardiac contractility, and systemic arterial blood pressure. Moderate to severe left ventricular failure in myocardial infarction resulting in pulmonary congestion and edema should not be treated with β-adrenergic blocking agents. ACE inhibitors, such as lisinopril, are usually started within the first 24 hours of infarction, usually following completion of fibrinolytic therapy. ACE inhibitors do not significantly change cardiac output or heart rate, but they are extremely useful in the treatment of heart failure and after myocardial infarction. In patients with unstable angina or myocardial infarction, short-acting calcium channel blockers, such as nifedipine, can increase the risk of adverse cardiac events and are therefore contraindicated. In patients with non-Q-wave MI, diltiazem can decrease the frequency of post-infarction angina and may be used.

A 76-year-old man with hypertension, coronary artery disease, and poorly-controlled left ventricular congestive heart failure presents with a 3-day history of insidious chest pain. Pain is made worse when he takes a deep breath in and when he coughs. He denies any relation of pain to position, activity, or food intake. He denies fever, chills, palpitations, sputum production, wheezing, abdominal pain, nausea, vomiting, diarrhea, or peripheral edema. His physical exam reveals a widespread friction rub upon inspiration, absent lung fremitus, and reduced lung sounds over the thoracic cavity. Question What finding would be anticipated upon further diagnostic testing? 1 Serum glucose to pleural ratio of greater than 1.0 on pleural fluid analysis 2 Pleural fluid N-terminal pro-brain natriuretic peptide levels greater than 1500 pg/mL 3 Lobar consolidations with air bronchograms on chest radiography 4 Hyperinflation, hyperlucency, and depressed diaphragms on chest X-ray 5 Pleural to serum LDH ratio of greater than 0.6

Answer: 2 Pleural fluid N-terminal pro-brain natriuretic peptide levels greater than 1500 pg/mL This patient's presentation is most consistent with a pleural effusion. This patient is demonstrating pleuritis, which is caused by damage and inflammation to the pleura of the lung. The most common cause of pleural effusion is left ventricular failure, which causes a transudative effusion. The characteristics of pleural fluid transudate are a yellow appearance, specific gravity of less than 1.016, absolute protein of less than 3 grams per 100 mL, a pleural protein to serum ratio of less than 0.5, a pleural to serum LDH ratio of less than 0.6, a serum to pleural glucose ratio of less than 1, and very low WBCs. A pleural fluid N-terminal pro-brain natriuretic peptide (NT-proBNP) >1500 pg/mL is virtually diagnostic of an effusion secondary to congestive heart failure. The earliest sign of pleural effusion on the frontal view of a chest radiograph is a blunting of the lateral costophrenic angle. A large free pleural effusion appears as a dependent opacity with lateral upward sloping of a meniscus-shaped contour. The diaphragmatic contour is partially or completely obliterated, depending on the amount of fluid (silhouette sign). Lobar consolidations with air bronchograms, patchy airspace opacities, or diffuse alveolar or interstitial opacities are common radiographic findings consistent with pneumonia. Hyperinflation, hyperlucency, and depressed diaphragms upon chest X-ray assessment suggest asthma. Increased lung volumes and increased retrosternal airspace may also be observed.

A 22-year-old man presents with a right groin bulge. During physical assessment, a single sac is found protruding just lateral to the epigastric vessels. Although the clinical scenario is highly suggestive of a hernia, what initial imaging study would help support this diagnosis? 1 Radiograph 2 Ultrasound 3 CT scan 4 MRI 5 Doppler study

Answer: 2 US Ultrasonography is a convenient, safe, noninvasive imaging study that can be used even at bedside in this patient scenario to help confirm this is a true hernia. It is an imaging technique that utilizes sonic energy, which is also termed a nonionizing type of energy, so it is safe for use in all patients, including pregnant and pediatric patients. Images are created by echoes or reflections of the ultrasound beam as it interacts with tissues of different properties. Ultrasound waves are greatly reflected by superficial, air-soft tissue, and bone-soft tissue interfaces, making its usage somewhat limited. Because the inguinal structures are superficial, a linear transducer ultrasound is very effective at helping confirm the diagnosis. Larger body habitus does have to be taken into consideration in terms of interference of the ultrasound results. Having the patient perform the Valsalva maneuver at various stages of the ultrasound can help identify if the hernia has some degree of being transient. The characteristic movement of the herniating tissues that will be visualized during an active ultrasound study usually helps confirm the diagnosis. A radiograph would not be appropriate in this scenario. Computed tomography (CT) is another imaging study that is commonly used to help assess hernias, but it may not be the first diagnostic study chosen. CT scans produce exquisitely adequate images of all intra-abdominal organs and the abdominal wall, and they can create these images quickly. Because of its superior anatomic detail, CT may also help detect subtle signs of complication within the hernia sac, including bowel obstruction, incarceration, strangulation, and traumatic wall hernias. MRIs accurately depict groin hernias, but they may not be as readily available, and they are much more expensive than ultrasonography and computed tomography. Doppler studies are reserved for suspected issues that primarily arise in the cardiovascular system.

A 28-year-old man presents with rectal bleeding. The patient has noticed blood with bowel movements 3 times. The blood is described as bright red in color and small in amount. He also complains of rectal pain, especially with passing hard stools. He has tried some over-the-counter hemorrhoid creams without relief. The patient admits episodic constipation. He denies dark, tarry stools, easy bruising, and prior episodes of rectal bleeding. He has not noticed blood in his urine or with brushing his teeth. He denies nausea, vomiting, diarrhea, fevers, and weight loss. He has no known medical conditions. Family history is negative for gastrointestinal disorders. Social history reveals he is in a heterosexual relationship, and he denies anal intercourse. On physical exam, his abdomen is normal. The anus has no visible protrusions or rash, but there is a very small erythematous and tender area that appears like a "paper cut" or crack in the skin. The patient experiences pain with digital rectal exam (DRE). No masses are noted in the rectal vault. Question What is the most likely diagnosis? 1 Anal cancer 2 Anal fissure 3 External hemorrhoids 4 Genital warts (condyloma acuminata) 5 Pilonidal disease

Answer: 2 anal fissure This patient most likely has an anal fissure. Fissures are small, painful lesions in which the skin has a "cut" or "torn" appearance. The fissure most likely occurred secondary to anal sphincter stretching and tearing with passing of large, firm bowel movements. Fissures can be associated with a small amount of bright red rectal bleeding. Anal cancer could present with anal bleeding, but it is very rare. Fissure or hemorrhoids are much more likely explanations for the bleeding in this patient. External hemorrhoids are a common condition and can be associated with anal pain, constipation, and bright red rectal bleeding. The physical exam differentiates hemorrhoids from fissures. Hemorrhoids are dilated blood vessels and appear as protrusions, either singly or several clustered together, such as a bunch of grapes. Genital warts (condyloma acuminata) can cause irritated anal lesions. They are caused by the human papillomavirus (HPV) and are not typically associated with anal pain, rectal bleeding, or constipation. The appearance is quite different, with a bumpy, "cauliflower-type" lesion. Pilonidal disease is associated with a sinus or abscess formation in the intergluteal cleft. It can produce pain, but it is not associated with rectal bleeding. Pilonidal disease would be easily differentiated in this patient by location.

A 62-year-old man presents with a 2-month history of worsening fatigue and shortness of breath. He has a past medical history of emphysema; it is attributable to his history of smoking 85 packs of cigarettes per year. The patient complains of nearly passing out while climbing the stairs in his house. He tells you that he feels like his heart races. He reports chronic shortness of breath and cough, but now he feels like his dyspnea is dramatically worse; he can no longer sleep in his bed. He has been trying to sleep propped up in a chair at night. He is also experiencing fatigue. He has gained about 15 pounds, and he notes that he can no longer lace up his shoes. He denies fever, chills, and chest pain. His cough produces some mucus, but no hemoptysis. His vitals are shown in the table. Weight 212 lb Height 69" Body mass index 31.3 Pulse 108 Blood pressure 140/88 Temperature 98.2°F Pulse oximetry 88% On physical exam, you see a man in mild respiratory distress; he is sitting upright and leaning forward, and he uses accessory respiratory muscles for breathing. The exam is significant for reduced air movement and mild rales bilaterally in the lungs, distended neck veins, mild tachycardia with prominent P2, lower extremity edema, and right upper quadrant abdominal tenderness with hepatomegaly. Question Based on this patient's history and physical exam findings, what intervention is the most appropriate next step? 1 Initiate broad-spectrum antibiotics. 2 Initiate oxygen. 3 Order echocardiogram. 4 Order ventilation-perfusion scan. 5 Recommend vigorous exercise program to reduce weight. 6 Refer to cardiopulmonary specialist.

Answer: 2 initiate oxygen This patient, who has a history of emphysema and cigarette smoking, has progressed into cor pulmonale, which is also known as pulmonary heart disease. The next intervention should be to initiate oxygen. This patient is in respiratory distress with low oxygen saturation. Many clinicians may overlook this simple step, but oxygen therapy should be a key component of every patient with cor pulmonale. Broad-spectrum antibiotics would be indicated if this patient's history suggested an infectious etiology for his symptoms; however, he denies fever and chills; he is afebrile on exam and has multiple cardiovascular symptoms that are inconsistent with a pulmonary infection. An echocardiogram and referral to cardiopulmonary specialist are both appropriate interventions for a patient with cor pulmonale, but these interventions should take place after oxygen is initiated. A ventilation-perfusion (VQ) scan is a reasonable test to consider when pulmonary embolism (PE) is on the differential. A PE might present with dyspnea, cough, and tachycardia, but this patient's cardiovascular symptoms are not explained by a PE. If the clinician wanted to be complete and order a VQ scan, it also should be done after oxygen is initiated. Vigorous exercise can worsen cor pulmonale, and patients should be advised against significant exertion. With appropriate supervision by a therapist and medical management, individuals with cor pulmonale may gradually increase activity tolerance. This patient's recent weight gain was likely due to fluid retention, and it can be managed with appropriate diuresis.

A 50-year-old Caucasian woman presents with an oval skin growth on her forehead. She noticed the growth 2 weeks ago. It gradually grew in size and now bleeds occasionally. She works as a secretary at a dentist's office. She has diabetes and hypertension treated with atenolol and metformin, respectively. On exam, there is an erythematous papule about 3 cm2 in size that bleeds on probing. Her BMI is 30. Biopsy of the lesion reveals squamous cell carcinoma (SCC) of the skin. What is a risk factor for SCC in this patient? 1. Occupation 2. Ethnicity 3. Female sex 4. Obesity 5. Atenolol

Answer: 2. Ethnicity Ethnicity is the correct response. Caucasians are at greater risk of SCC than other ethnicities. Blue or green eyes and light-colored hair are also considered risk factors. Type I and type II skin, history of sunburn or indoor tanning, smoking, and immunosuppression are other risk factors. Her indoor occupation is not a risk factor, as there is lesser sun exposure than outdoor occupations. SCC is more common in men than women (2:1 ratio) due to greater cumulative sun exposure during their lifetime. Obesity and atenolol are unrelated to SCC.

A 61-year-old man with known cirrhosis presents with a 1-week history of "puffy" ankles and increased shortness of breath. A week prior to symptom onset, he was traveling while on vacation; he engaged in walking, sightseeing, and eating out. He admits more dyspnea with lying down and with increased exertion. His shoes feel snug, and he notes a definite line from wearing socks. The patient denies chest pain, leg pain, fevers, claudication, nausea, headache, lethargy, and hemoptysis. His past medical history is remarkable for the cirrhosis and a history of alcoholism. He is awaiting a liver transplant. He had a liver biopsy but no other surgeries. He takes no medications, has no allergies, and has had no alcohol for 9 months. He lives with his wife, works as an electrician, and smokes 1 pack of cigarettes per day. Vitals are normal, including oxygen saturation. On physical exam, the patient appears in no acute distress and with normal mental status. His physical exam is remarkable for mild jugular venous distention, 2-3+ edema in lower extremities, and mild dullness to lung percussion. No hepatomegaly or ascites is noted. This patient's laboratory results are shown in the chart. Complete blood count (CBC) Normal Chest X-ray Mild bilateral effusions Urinalysis Normal Beta-natriuretic peptide (BNP) Normal Complete Metabolic Panel (CMP) Abnormal as indicated below Test Result Normal range Sodium 126 mmol/L 134-144 Potassium 3.4 mmol/L 3.4-4.9 Chloride 100 mmol/L 100-109 HCO3 26 mmol/L 20-31 Glucose 98 mg/dL 70-99 Bun 18 mg/dL 43299 Creatinine 1.1 mg/dL 0.6-1.2 Calcium 8.8 mg/dL 8.8-10.5 Albumin 2.6 g/dL 3.5-5.0 Total Protein 5.9 g/dL 6.4-8.2 AST (SGOT) 112 U/L 15-37 ALT (SGPT) 108 U/L 15827 Alk Phosphatase 158 U/L 50-136 Total Bilirubin 1.9 mg/dL 0.1-1.2 eGFR >60 mL/min/1.73m2 >60 Osmolality 265 mmol/kg H2O 285-293 Question What is the most appropriate pharmaceutical treatment for this patient's fluid and sodium status? 1. Dexamethasone 2. Furosemide 3. Hypertonic saline 4. Sodium phosphate 5. Ursodiol

Answer: 2. Furosemide This patient is in a hypervolemic hypotonic hyponatremic state. The hypervolemia can be identified primarily by history (edema and dyspnea) and physical exam (jugular venous distention, edema, and pulmonary effusion). The hypotonia (low serum osmolality) and hyponatremia most likely are due to a "dilution" effect, in which this patient's overall fluid status is high, but the sodium levels are normal or even elevated. This patient has hypoalbuminemia (common with chronic liver disease) and increased extracellular fluid, which is causing his mild pulmonary effusions and dyspnea. The most appropriate treatment for this patient is to begin intravenous or oral diuresis, with a diuretic such as furosemide. Diuretics should be used with caution in patients with cirrhosis, but his pulmonary edema must be addressed. Dexamethasone is a steroid medication, which may be helpful in reducing inflammation. Steroids are used in a great number of medical conditions, but dexamethasone would not have a direct role in treating this patient's hypervolemia and hyponatremia. Hypertonic saline is an intravenous solution with a sodium chloride concentration greater than the physiologic 0.9%. It may come in a 3% and 5% concentration. As noted above, total body sodium is often actually high in patients with hypervolemic hyponatremia. Giving excess IV fluids with excess sodium would worsen his edema and his condition overall. Sodium phosphate is used as a bowel preparation prior to procedures. It draws fluid into the colon via osmotic action and has no role in treating serum sodium abnormalities. Ursodiol alters bile cholesterol composition and may be used for primary biliary cirrhosis and gallstone dissolution and prevention. With use for primary biliary cirrhosis, ursodiol is thought to reduce the buildup of toxic bile acids, limiting further damage to the liver. At presentation, we do not have in-depth details of this patient's hepatitis (although it was likely due to chronic alcoholism), so the use of ursodiol should be left up to his gastroenterologist managing his chronic condition. The primary care provider should address the fluid/sodium abnormality now, which ursodiol will not do.

32-year-old man presents due to low back pain. He reports a 3-year history of chronic low back pain that progressively worsened. He notes a lot of stiffness with any movements that involve his low back. He is especially stiff upon morning awakening and now has to ask his wife to help him get out of bed. He denies a history of trauma. He works in retail, with some bending and light lifting duties. He has taken acetaminophen repeatedly—even exceeding recommended doses—with no relief. He knows his paternal grandfather had some type of serious arthritis in his back, but he does not know the diagnosis. Other than his chronic back pain, the patient reports that he is in good health with no chronic conditions. He takes no regular medications, except for the analgesics mentioned. He has no allergies and has never had surgery. On physical exam, some limited range of motion is observed in his back and hips, and the patient reports discomfort with the motions. Otherwise, his exam is normal, with no visible edema, muscle wasting, or bony abnormality. You obtain a plain radiograph, and the report indicates a classic "bamboo spine" appearance and some evidence of sacroiliitis. His sedimentation rate and c-reactive protein levels are elevated. Question What medication would be an appropriate first-line treatment for this patient's condition? 1 Acetylcysteine (Mucomyst) 2 Allopurinol (Aloprim) 3 Celecoxib (Celebrex) 4 Cyclobenzaprine (Flexeril) 5 Milnacipran (Savella)

Answer: 3 Celecoxib This patient's presentation is consistent with a diagnosis of ankylosing spondylitis (AS), a progressive inflammatory arthritis that primarily affects the axial skeleton. NSAIDs are the primary first-line treatment, so celecoxib (Celebrex) would be reasonable. Cyclooxygenase-2 (COX-2) inhibitors, such as celecoxib, have not been necessarily shown to have better efficacy, but are suggested to have fewer gastrointestinal side effects than traditional NSAIDs. Regular use of NSAIDs is recommended. Other potential treatments include medications such as methotrexate, sulfasalazine, and newer "biologics," like infliximab, etanercept, and adalimumab. Acetylcysteine (Mucomyst) is a medication used for acetaminophen overdose. The patient's current symptoms do not suggest acute acetaminophen toxicity. Allopurinol (Aloprim) is a medication used primarily for gout. While gout can cause joint pain, this patient's presentation is not consistent with gout. Cyclobenzaprine (Flexeril) is an older muscle relaxant medication. Although the patient reports "stiffness," his symptoms are due to joint damage, not muscle spasm. This medication would not help his AS. Milnacipran (Savella) is a medication approved for fibromyalgia. While chronic pain is found in both AS and fibromyalgia, none of this patient's diagnostic studies are consistent with fibromyalgia.

A 72-year-old man presents due to worsening shortness of breath, orthopnea, and chest pain; symptoms have been occurring for the past few weeks. The patient admits to some chronic heart problems as well as fatigue, dyspnea, and a non-productive cough, but he feels like symptoms have worsened recently. He denies fever, chills, and a productive cough. On physical exam, the man has mildly increased respiratory effort, but he does not appear in distress. He is barrel-chested. His breath sounds are diminished bilaterally, with dullness to percussion over right and left lower lungs. No pleural friction rub is noted. On cardiovascular exam, an S3 gallop and mild tachycardia (110 bpm) are noted. Clubbing of the fingers, dependent edema in the lower extremities, and jugular venous distention are also noted. His cardiac enzymes and electrocardiogram demonstrate no acute cardiac pathology. Pleural fluid and cardiomegaly are found on the chest X-ray, and a thoracentesis is performed. The pleural fluid is generally clear in color, testing negative for chylomicrons and triglycerides. It has low levels of red blood cells, white blood cells, protein, and lactate dehydrogenase (LDH). Question What should be recommended as most useful for this patient as a tertiary level of preventive medicine? 1 Administration of the influenza vaccine 2 Checking a prostate-specific antigen (PSA) to detect prostate cancer at an early stage 3 Control of his heart failure to prevent pulmonary complications 4 Daily low-dose aspirin 5 Calcium supplementation to reduce risk of fractures

Answer: 3 Control of his heart failure to prevent pulmonary complications Preventive medicine is often classified into primary, secondary, and tertiary methods. Primary methods are meant to prevent disease occurrence altogether. Secondary methods are intended to detect disease at a very early stage, before complications. Tertiary methods may occur after the patient already has some significant illness, and they are aimed at preventing further complications from the disease. In this patient's case, control of his heart failure to prevent pulmonary complications is a recommended tertiary preventive medicine. This patient's chronic heart failure has caused a pleural effusion. Better control of his heart condition will prevent pulmonary complications. Administration of the influenza vaccine is a recommended primary method of preventive medicine for this patient because it prevents the illness altogether. It is recommended for almost all adults, and it is especially recommended for the patient in this case, but it is not classified as tertiary prevention. Checking a prostate-specific antigen (PSA) to detect prostate cancer at an early stage is considered a secondary level of prevention; it is aimed at detecting the illness at a very early and treatable stage. PSA screening for prostate cancer is controversial, and because this patient is much more likely to succumb to complications from his heart failure and pleural effusion than prostate cancer, it should not be recommended at this time. Daily low-dose aspirin has been often recommended as primary prevention of stroke and myocardial infarction in high-risk individuals as well as secondary prevention. While aspirin therapy may offer some mild benefit to this patient (who has a history and physical consistent with chronic congestive heart failure), it is a not a top priority. The provider should not recommend calcium supplementation to reduce risk of fractures in this patient. Calcium supplementation has recently been linked to an increased risk of cardiovascular events. Except for specific conditions, widespread calcium supplementation is not recommended. It would also be a primary prevention technique, intended to prevent bone loss and fracture before disease was present.

A 54-year-old man presents with chest pain. He has a past medical history of hypertension and diabetes mellitus. The pain is located in the middle of his chest and radiates to his jaw. The pain began about 20 minutes ago, and he rates the pain as a 10 on a 0-10 point scale, with 10 being the worst pain he has ever felt. He has had 3 similar episodes, but they have always resolved after 5 minutes or so of rest. He has smoked 1 pack of cigarettes a day for the past 36 years. He drinks 2 or 3 beers on Friday nights. Review of systems (ROS) is positive for diaphoresis, acute dyspnea, and impending doom. ROS negative for fever, chills, and malaise. Physical exam shows an obese, middle-aged man in moderate distress. BP is 126/80 mm Hg, pulse is 100, and respirations are 26. Heart and lung exams are normal, except for tachycardia and tachypnea. He has no pedal edema. Question What aspect of the patient's history is the most influential nonmodifiable risk factor for an acute myocardial infarction? 1 Alcohol use 2 Cigarette smoking 3 Diabetes mellitus 4 Hypertension 5 Obesity

Answer: 3 Diabetes mellitus Diabetes mellitus (DM) is an independent risk factor for atherosclerosis. The risk of myocardial infarction (MI) in a patient with diabetes is the same risk as someone without diabetes who has had a previous MI. The risk of death from cardiac events is also the same between the two groups. Other important contributors to atherosclerosis and ischemic heart disease, including dyslipidemia, cigarette smoking, obesity, and hypertension, contribute to the risk of MI, but DM is a greater risk factor when evaluated independently. Alcohol consumption may have a protective effect. People who drink moderate amounts of alcohol reduce the risk of cardiovascular events. Patients with diabetes mellitus should be advised to stop smoking and aggressively control other risk factors such as glucose, hypertension, and dyslipidemia in order to reduce the risk of ischemic heart disease.

A 19-month-old boy presents with a 3-day history of fever, irritability, and poor feeding. He also began to drool and his mother can hardly get him to eat anything. His past history is significant for recurrent ear infections. He had tympanostomy tubes placed about 1 month ago. On examination, he is febrile with a temperature of 39.8°C (103.6°F). He appears unwell and is uncooperative and irritable when touched. He is noted to be drooling and has several vesicles and shallow ulcers on his tongue and palate. The ulcers have a yellowish base with surrounding erythema. There are no lesions on any other areas of his body. Question What would be the most appropriate treatment for this child? 1 Amoxicillin 2 Oseltamivir 3 Acyclovir 4 Prednisone 5 Nystatin

Answer: 3 acyclovir This child has symptoms and signs consistent with herpetic gingivostomatitis. Acyclovir is recommended for treatment of herpetic gingivostomatitis and leads to faster resolution of symptoms. Symptomatic treatment with oral analgesics and cold soothing foods, such as ice pops, is also helpful. Amoxicillin would not be used to treat a viral infection. Antibiotics may be indicated if there is evidence of secondary bacterial infection. Oseltamivir is an antiviral medication used to treat and prevent influenza A and B; it does not treat gingivostomatitis. Systemic steroids (prednisone) would likely worsen a viral infection. Nystatin is an antifungal and would be indicated for the treatment of fungal infections like oral thrush.

A 52-year-old woman presents with a 1-year history of bilateral shaking in her hands. The shaking tends to worsen when she is using her hands; her symptoms improve when she drinks a glass of wine on the weekends. Her 82-year-old mother also has a similar shaking in her hands that developed when she was in her 50s. Question What is the most likely diagnosis? 1 Alcohol withdrawal 2 Parkinson's disease 3 Essential tremor 4 Huntington's disease 5 Multiple sclerosis

Answer: 3 essential tremor' Essential tremor (or benign tremor) typically involves the hands or head, but not the legs. The cause is unknown, but it may be inherited. Patients note that the tremor develops when moving the hands. The symptoms may worsen in times of stress, and they may be alleviated by alcohol intake. Alcohol withdrawal may be associated with a tremor, but this is unlikely, given the duration of the tremor. The tremor of Parkinson's disease is typically a resting tremor; this patient is not experiencing other symptoms of Parkinson's disease (e.g., rigidity, bradykinesia, postural instability). Huntington's disease may be associated with abnormal movements and is inherited in a familial matter, but Huntington's disease is typically fatal within 20 years of onset. This patient notes that her mother has been suffering from her tremor for close to 30 years. Multiple sclerosis is not typically associated with tremor.

A 67-year-old woman presents; according to her, her husband says she never listens to anything he says. The patient states that occasionally she has to ask people to repeat themselves when sitting to her right. She denies any dizziness, headaches, or visual disturbances. Her current medication is furosemide. On physical examination, the Weber test reveals lateralization to the left ear. On the left ear, air conduction lasted for 15 seconds and bone conduction lasted 10 seconds. On the right ear, air conduction lasted for 22 seconds and bone conduction lasted 10 seconds. Question What do you suspect is the cause of this hearing loss? 1 Cerumen impaction 2 Otosclerosis 3 Ototoxicity 4 Meniere's disease 5 Middle ear effusion

Answer: 3 ototoxicity Ototoxicity secondary to furosemide is the correct answer. Loop diuretics can cause sensory hearing loss, as evidenced by this patient's physical exam finding of lateralization to the good ear and air conduction slightly longer than bone conduction. The Rinne test should reveal an air-bone conduction ratio of 2:1. Cerumen impaction will cause a conductive hearing loss with the lateralization to the affected ear and a negative Rinne test. Otosclerosis typically will result in conductive hearing loss. Meniere's Disease is incorrect because the patient does not exhibit any vertigo or tinnitus. Middle ear effusion is incorrect because it would cause a conductive hearing loss.

A 12-year-old girl presents with a 1-week history of a rash on her trunk. The patient has not been ill or exposed to anyone ill. On examination, there are scattered lesions on her trunk; they look like they form a Christmas tree. The girl states that the lesions are itchy at times, but they are generally not bothersome. Question What diagnosis is suspected? 1. Tinea corporis 2 Seborrheic dermatitis 3 Pityriasis rosea 4 Tinea versicolor 5 Viral exanthem

Answer: 3 pityriasis rosea Pityriasis rosea is a rash that typically occurs in girls/women and is located truncally. The lesions will generally form in the shape of a Christmas tree. Tinea corporis would be causing an annular pruritic lesion. Seborrheic dermatitis would most likely have scaly patches associated with the lesions. Tinea versicolor would yield hypopigmented or hyperpigmented lesions. The patient has not had any sick contacts or been ill herself, so a viral exanthem can be ruled out.

A 19-year-old Caucasian man has come to see you as the last patient of the day. He presents with sudden onset of severe shortness of breath. He states that he has been an avid basketball player all his life and was practicing about 4 hours prior to his visit when he experienced sudden chest pain and immediate shortness of breath that is still bothering him currently. He describes the chest pain in the middle of the chest, more so on the right anterior side. The patient admits to smoking half a pack of cigarettes daily. Physical examination reveals a tall, thin, well-developed man in mild distress. The only other abnormalities discovered are mild tachycardia (120 beats per minute) and diminished breath sounds in the posterior right lower lobe. Question Based upon the examination so far, what is the most likely diagnosis? 1 Pneumonia 2 Pulmonary embolism 3 Pneumothorax 4 Myocardial infarction 5 Asthma

Answer: 3 pneumothorax The clinical scenario is highly suggestive of a primary pneumothorax. Components that lead to this diagnosis include unilateral sudden chest pain with dyspnea and minimal physical exam findings, in this case, the mild tachycardia and diminished breath sounds. Pneumothorax is more commonly found to occur in tall, thin, male patients between ages 10 and 30; it is more common in those who smoke. Diagnosis is usually confirmed with chest radiograph. Treatment is dependent on the size and severity of symptoms: a pneumothorax that is <15% of a hemothorax and is stable will typically resolve on its own without intervention. Reoccurrence is as high as 30%. Pneumonia would more commonly present with a substantially high fever, tachypnea, and nonproductive or productive cough. Physical examination would reveal bronchial breath sounds or even crackles. Although the patient is a smoker and this is considered a risk factor for community-acquired pneumonia, this is highly unlikely to be the diagnosis. Although patients who have a pulmonary embolism may experience dyspnea and chest pain, they may also have hemoptysis, as well as syncope, and may present with extreme anxiety. Patients with pulmonary embolism often have a strong risk factor for hypercoagulability, including but not limited to history of thromboembolism, known genetic hypercoagulable state, and one or more components of Virchow's triad. Myocardial infarction is not likely because of the onset of chest pain occurring during activity; many times the pain of an infarction occurs at rest, usually first thing in the morning. Pain from a myocardial infarction may be similar to angina pain in location and radiation, but it intensifies rapidly to the point where the pain is not even relieved by opioids. This patient is also in a highly unlikely age group to suffer from an acute myocardial infarction. Asthma is also not likely as the diagnosis because this is generally a chronic disorder that shows symptoms of airflow obstruction during any situation that affects the airways, such as triggers, exercise, or infection. It is slightly more common to be diagnosed in male patients under 14. Symptoms are present during active exacerbated episodes and may include wheezing, chest tightness, or coughing, but they can be extremely variable from patient to patient.

A 55-year-old woman presents with a 2-month history of gross hematuria. She states she has no pain with urination, but the hematuria is persistent. On questioning, she states that she does have some progressively worsening left flank pain and persistent back pain over the past month. The pains are not debilitating, but they are nagging. She has no chronic medical problems. She admits to a 50 pack-year smoking history, and she states she is currently retired from her job as a teacher. Vital signs are within normal limits and physical exam reveals a left side abdominal mass. Urine dipstick only shows too numerous to count RBCs; urine cultures are negative. Question What is the most likely diagnosis? 1 Acute cystitis 2 Bladder cancer 3 Renal cell carcinoma 4 Urethritis 5 Ureteral calculi

Answer: 3 renal cell carcinoma Hematuria is the most common presenting sign of urinary tract cancer. Silent or painless hematuria suggests tumor or renal parenchymal disease. Renal cell carcinoma can present with flank pain, hematuria, persistent back pain, and an abdominal mass. It can also be found incidentally on CT scan, so the clinical picture points to cancer with renal cell carcinoma as the most likely diagnosis. Smoking is a risk factor for renal cell cancer. RCC is more common in men than women (2:1), and it has a peak incidence in individuals in their 50s. Bladder cancer often presents with gross hematuria, but it is most typically painless. Bladder cancer can present with pain if the cancer is blocking the ureter. The left flank pain and back pain are more characteristic of RCC. Bladder cancer is the second most common urologic cancer, and the mean age at diagnosis is 65. It is more common in men than women (2.7:1) and 98% of primary bladder cancers are epithelial malignancies (majority urothelial cell carcinomas). Acute cystitis typically presents with irritative voiding symptoms (frequency, urgency, dysuria) and suprapubic discomfort in addition to possible hematuria. Urinalysis will show pyuria, bacteriuria, and varying degrees of hematuria. Urine cultures will show specific organisms. Urethritis is inflammation of the urethra that presents with urethral discharge, dysuria, and itching. Urethritis is most often caused by an STD. This patient does not have a history of unprotected sexual intercourse or any other symptoms that would indicate this diagnosis. Ureteral calculi can present with hematuria; it also typically presents with flank or abdominal pain, but not abdominal mass. If the stone is in the ureter, it often causes some hydroureter with or without hydronephrosis, both of which cause some pain or discomfort. The patient has no past history of forming stones.

A 22-year-old woman presents with an 8-month history of amenorrhea. The patient is also experiencing backaches, headaches, and acne. Physical examination reveals a female patient with a moon-shaped facies, multiple purple striae, and significant central obesity (body mass index of 36). 1. Type 2 diabetes 2. Hypothyroidism 3. Cushing syndrome 4. Hyperthyroidism 5. Diabetes insipidus

Answer: 3. Cushing Syndrome Explanation: This patient likely is suffering from Cushing syndrome, also known as hypercortisolism. Consequences of excessive levels of circulating cortisol, no matter the etiology, will lead to signs and symptoms such as central obesity, thin extremities, a moon face, a buffalo hump, supraclavicular fat pads, protuberant abdomen, oligomenorrhea, and amenorrhea; in men, erectile dysfunction is sometimes seen. Backaches, headaches, hypertension, acne, purple striae, and impaired wound healing may also be found in these patients. Type 2 diabetes is caused by insulin resistance in tissues to circulating endogenous insulin. Eventually, insulin is unable to be produced by the pancreatic islet cells, and an exogenous supply is required to prevent significant hyperglycemia and its related co-morbidities. Features of these patients include central obesity (specifically visceral obesity), with less fat noted on extremities. Patients will have a history or evidence of acanthosis nigricans, eruptive xanthomas on flexor surfaces, skin tags, chronic skin infections, frequent candidal vulvovaginitis in women, and erectile dysfunction or balanoposthitis in men. Both hypothyroidism and hyperthyroidism are related to dysfunction of the thyroid gland; hypothyroidism is due to failure of the thyroid gland itself or deficient supply of pituitary TSH (thyroid stimulating hormone). Symptoms characteristic of hypothyroidism include weight gain, fatigue, lethargy, depression, weakness, menses irregularities, athralgias, muscle cramps, cold intolerance, dry skin, and constipation; other signs may include thinning hair, brittle nails, bradycardia, or even peripheral edema. Hyperthyroidism is the disease state in which there is an excessive serum level of T3 and or T4 along with suppressed TSH levels. Signs and symptoms may include restlessness, nervousness, heat intolerance, increased sweating, weight loss, palpitations, atrial fibrillation, exophthalmos, or (depending on the severity) even pretibial myxedema. Diabetes insipidus (DI) is an antidiuretic hormone deficiency. Signs and symptoms of DI include an extreme, unquenchable thirst with a very specific craving for ice water. Patients also will have significant polyuria or hypernatremia.

68-year-old woman presents with a several-month history of feeling dizzy and light-headed. She admits brief syncope once. These episodes often occur first thing in the morning when she gets out of bed and often after dinner. She admits some general weakness but otherwise feels well. She denies seizures, headaches, numbness, paresthesias, and gait or balance disturbances. The patient did some routine labs for a health fair last month and reports all values returned normal. The patient's past medical history is noncontributory. She is menopausal, takes no medications, has had no surgeries, and has no allergies. She is retired, lives with her husband, and does not exercise regularly. On physical exam, her blood pressure is taken both lying supine and then again 3 minutes later while standing upright. Readings are 110/78 mm Hg supine and 82/63 mm Hg standing. Mucus membranes are moist, and skin turgor is good. The patient's physical exam, including cardiovascular, pulmonary, and neurological, are normal. An electrocardiogram is performed and is normal. Question Assuming the workup does not reveal an underlying cause of this patient's condition, what non-medication intervention is the most appropriate for her? 1 Daily magnesium supplementation 2 Implantation of spinal cord stimulator 3 Liberal salt and fluid intake 4 One serving of red wine daily 5 Regular aerobic exercise

Answer: 3. Liberal salt and fluid intake This patient is presenting with orthostatic hypotension (OH), which is defined by a supine-to-upright positional drop in systolic blood pressure by ≥20 mm Hg and/or a drop in diastolic blood pressure by ≥10 mm Hg. Common symptoms are lightheadedness, dizziness, and syncope, which may occur more frequently with position changes, such as getting up and out of bed. Recommended interventions for OH include liberal salt and fluid intake, physical maneuvers such as leg crossing, and bending over to increase blood pressure and prescription fludrocortisone. Daily magnesium supplementation is not recommended for OH. Magnesium deficiency may occur with acute or chronic GI fluid loss, alcohol abuse, and several other conditions. Magnesium deficiency may be asymptomatic or present similarly to hypocalcemia and can readily be detected on a serum magnesium level. Implantation of a spinal cord stimulator may be recommended for chronic pain conditions, such as low back pain and complex regional pain syndrome. Spinal cord stimulators do not have a role in increasing blood pressure and would not help a patient with OH. One serving of red wine daily has sometimes been promoted for overall heart health due to antioxidants and the compound resveratrol. However, individuals with OH are advised not to consume alcohol, as it may worsen their condition. Regular aerobic exercise seems to be a reasonable recommendation for nearly all capable patients, with benefits for weight and cardiovascular health. However, the role of exercise for OH is complicated. Exercise can cause changes in the blood pressure, and for this patient who has not had a regular exercise routine, initiating one may worsen her OH. With careful supervision of the regimen, exercise may improve OH symptoms on a case-by-case basis.

A 36-year-old woman presents with a tibial fracture following a motor vehicle accident. She is admitted to the hospital. Overnight, she develops increased pain despite maximal doses of IV narcotics. What is the most reliable early physical exam finding for her condition? 1 Absent peripheral pulses 2 Pallor of the involved limb 3 Pain out of proportion to exam findings 4 Paresthesia of the involved limb 5 fever

Answer: 3. Pain out of proportion to exam findings The patient is suffering from compartment syndrome. Compartment syndrome occurs when pressure increases inside of a muscle compartment. It typically develops a few hours after soft tissue injury. Pain is typically out of proportion to exam findings and is unresponsive to opioid analgesia. The symptoms of compartment syndrome have classically been described by the 5 Ps: pain, pulselessness, paresthesia, pallor, and poikilothermia, but not all patients will develop this constellation of symptoms. Absent peripheral pulses may be present in a patient with compartment syndrome, but arterial circulation may remain intact until late in the presentation of compartment syndrome; a patient with intact peripheral pulses may still be suffering from compartment syndrome. Pallor of the involved limb may be present in a patient with compartment syndrome, but it is a late finding that does not develop until arterial circulation is affected. Paresthesia of the involved limb is another late finding of compartment syndrome. It develops when nerve ischemia is present. Fever is not a classic symptom of compartment syndrome.

A 12-year-old pre-menarcheal girl is referred to you following a screening for scoliosis at school by the school nurse. The nurse used a scoliometer that revealed a reading >5° and felt radiographs were needed at this point. The girl is totally asymptomatic of any back pain or motion or activity limitations. You order a standing AP and lateral long cassette spine and use the Cobb method to measure your curvatures. You measure and find a thoraco-lumbar single curve of 20° with a Risser stage of 3. What is recommended for this patient? 1. Brace immediately 2. Recheck radiographs in 1 year 3. Repeat radiographs in 3 months 4. Refer for spinal surgical stabilization. 5. No need for further follow-up since she is now skeletally mature.

Answer: 3. Repeat radiographs in 3 mos Treatment for scoliosis is partly based on the physiologic maturity of the skeleton (Risser classification) and the curve. For progressive curves in a patient with Risser stage of 3, it is recommended that you repeat radiographs in 3 months. 1 year is too long to wait in an actively growing child. Bracing is reserved for curves >30° in a patient with Risser stage of 3. Surgical referral is not needed until idiopathic curves reach 50° or for curves of 40-50° that are likely to progress. This patient should be followed up with every 3 months until skeletally mature. Risser Classification (level of ossification and fusion of the iliac crest apophyses): stage 0: no ossification center at the level of iliac crest apophysis stage 1: apophysis under 25% of the iliac crest stage 2: apophysis over 25-50% of the iliac crest stage 3: apophysis over 50-75% of the iliac crest stage 4: apophysis over >75% of the iliac crest stage 5: complete ossification and fusion of the iliac crest apophysis Degree of Curve and Therapy 0-25° of curve Observe 24-40° of curve Brace >40° of curve (immature) Surgery >50° of curve (mature) Surgery

A 70-year-old Hispanic man with a past medical history of hyperlipidemia presents with recurrent chest pain that has been occurring over the past month. This pain is provoked with activity, especially of his upper torso and left arm. The pain is described as sharp and generally located in the left pectoral area. He denies any weight changes, fever, or chills. He also denies cough, wheezing, pleurisy, calf pain, wheezing, vomiting, diarrhea, changes in bowel habits, cigarette, drug or alcohol use, sick contacts, and travel. Physical exam reveals an elderly Hispanic man in no acute distress. Vital signs, skin, cardiopulmonary, abdominal, and peripheral vascular exams are found to be within normal limits. Question What additional historical or physical exam finding would suggest a non-ischemic etiology of this patient's chest pain? 1 Pain associated with anxiety, nausea, shortness of breath, and syncope 2 Pain relief upon administration of nitrate drugs 3 Associated history of diabetes mellitus and hypertension 4 Chest pain that is reproduced with palpation 5 Quality of pain described as a pressing, tightness, or heaviness

Answer: 4 Chest pain that is reproduced with palpation The additional finding of chest pain reproduced with palpation suggests chest wall pain. Patients with chronic stable angina, representing a fixed coronary obstruction, typically present with a precordial, retrosternal, or anterior chest pain that is constricting, squeezing, or suffocating. Episodes are commonly precipitated by physical exertion, exposure to cold, and emotional stress. Pain may radiate to the left shoulder, jaw, arm, or other parts of the chest. Chronic stable angina is relieved quickly by rest or use of nitroglycerin. Major risk factors for CAD include cigarette smoking, hypertension, hyperlipidemia, diabetes, advanced age, abdominal obesity, and a sedentary lifestyle. Acute Coronary Syndrome (ACS), which includes unstable angina, non-STEMI, and STEMI, usually consists of pain of over 20 minutes duration, severe (than previously experienced), of a new onset. Pain is described as crushing, constricting, suffocating, and associated with chest pressure. Its location and radiation pattern are similar to stable angina. Women may complain of atypical patterns of chest pain, while elderly patients may complain of shortness of breath. With STEMI, nausea and vomiting may occur, along with fatigue, weakness, anxiety, restlessness, feelings of impending doom, skin pallor, coolness, and moistness. Hypotension and shock may also occur in MI.

Patients suffering from asthma, no matter the classification, should be prescribed what medication? 1 Inhaled corticosteroid 2 Inhaled long-acting β2-agonist 3 Leukotriene receptor antagonist 4 Inhaled short-acting β2-agonist 5 Omalizumab

Answer: 4 Inhaled short acting B2 agonist Inhaled short-acting β2-agonist (SABA) is the appropriate answer. SABA is considered a quick-relief medication that acts by relieving and reversing acute airflow obstruction. Whether the patient is diagnosed with intermittent or persistent asthma, SABA is the drug chosen to be used for acute symptomatic periods and is typically instructed to be used on an as-needed basis.Patients are explicitly educated on the "Rule of 2s" in relation to the use of SABA: if using SABA >2 days a week, if waking up at night due to asthma or Patients are explicitly educated on the "Rule of 2s" in relation to the use of SABA: if using SABA >2 days a week if waking up at night due to asthma or cough more than 2 times in a month if having to refill their prescribed SABA more than 2 times a year this is highly suggestive that there is inadequate control and that a step up treatment plan needs to be made in terms of asthma management. SABA can be administered via an inhaler or nebulizer machine. Inhaled corticosteroids, long-acting β2-agonists, and Leukotriene receptor antagonists are used in varying degrees, dosages, and combinations for long-term control (maintenance) of asthma. Maintenance medications are taken daily whether or not the patient is symptomatic. Depending on the medication, the route of administration could be via inhalation or an oral tablet. Omalizumab is a subcutaneous injection indicated for patients with moderate to severe persistent allergic asthma caused by year-round airborne allergens. This injection is given every 2-4 weeks in the provider's office.

A 6-year-old boy presents with his mother who has gradually progressive concerns regarding his behavior. The mother notes that her son does not interact or play with other children in school. He behaves indifferently to loved ones entering and exiting rooms. He screams and cries inconsolably at times upon exposure to loud sounds. He reacts angrily to bright lights and when family members hug him. He appears to be quieter than his peers. He has difficulty speaking in the first person when trying to communicate his desires. Physical exam reveals a 6-year-old boy with poor eye contact who is observed turning the water faucet on and off incessantly in the office, intently focused on the water stream. When asked to point to a particular object in the room, the patient is unable to do so. Question What agent would be most useful in controlling this patient's irritability and repetitive behavior? 1 Haloperidol 2 Methylphenidate 3 Fluoxetine 4 Clonazepam 5 Risperidone

Answer: 5 risperidone This patient's most likely diagnosis is autism spectrum disorder. Neuroleptic medications, particularly the atypical neuroleptics, are effective in decreasing stereotypical behavior and agitation. The second-generation antipsychotic agents risperidone and aripiprazole provide beneficial effects on challenging and repetitive behaviors in children with autism spectrum disorder, although these patients may experience significant adverse effects. Risperidone and aripiprazole have been approved by the United States Food and Drug Administration (FDA) for irritability associated with autistic disorder. Additionally, the second-generation antipsychotic agent ziprasidone may help to control aggression, irritability, and agitation. Haloperidol is a first-generation antipsychotic medication not indicated in the treatment of autism spectrum disorder. Hyperactivity often improves with stimulants such as methylphenidate. Serotonergic drugs, such as fluoxetine, are reportedly beneficial for improving behavior in autistic disorder. SSRIs are widely prescribed for children with autistic disorder or a related condition. These agents are used off-label to help with intractable repetitive behaviors (e.g., compulsion), but not with irritability, rage, or agitation. At this time, benzodiazepines are not used to treat autistic manifestations in humans; improved social interaction, decreased repetitive behaviors, and better spatial learning were observed in mice treated with clonazepam.

A 53-year-old man was recently diagnosed with a pituitary adenoma. He has been experiencing excessive thirst and large volume polyuria, as well as headaches and an increase in nocturia from once nightly to 3-4 times nightly. Question What medication should be the first-line treatment option given to him? 1 Hydrochlorothiazide 2 Indomethacin 3 Amiloride 4 Desmopressin acetate 5 Glucophage

Answer: 4 desmopressin acetate Desmopressin acetate (DDAVP) is used as the first-line treatment in patients with central or primary diabetes insipidus (DI). Arginine vasopressin (AVP), or antidiuretic hormone, is produced by the pituitary gland and works on the renal tubules within the kidney to concentrate the urine and reduce water loss. DI occurs when there is a deficiency in AVP, and these patients produce large amounts of dilute urine. As a result, patients can experience excessive thirst, polyuria, nocturia, headache, dehydration, and disturbances in vision. Central or primary DI can be diagnosed with 24-hour urine collection or vasopressin challenge test. Once the diagnosis is established, the first-line treatment is usually desmopressin acetate, which is a synthetic form of AVP. It has a longer antidiuretic action than AVP and does not constrict smooth muscle. DDAVP is available in oral, nasal, and injectable preparations. While being treated, it is important to control fluid balance in order to prevent dehydration, monitor the patient's weight daily, and monitor electrolytes. Hydrochlorothiazide can be used in cases of nephrogenic DI rather than central DI and as a treatment for hypertension. Nephrogenic DI occurs in the presence of normal AVP production by the pituitary gland with insensitivity to AVP in the kidney. There is a disorder in the renal tubular function that results in the inability to respond to the presence of AVP. In these cases, hydrochlorothiazide can be given in a 25 mg dose either once or twice daily. Indomethacin is a non-steroidal anti-inflammatory that is typically used in the treatment of osteoarthritis, rheumatoid arthritis, or ankylosing spondylitis. It can also be used in the treatment of nephrogenic DI, however, because indomethacin inhibits prostaglandins, which reduce urine output by decreasing the renal blood flow. This would not be effective in the treatment of central DI. Amiloride is often added to HCTZ to increase the diuretic effects, but this is a treatment course that is used in nephrogenic DI and not central DI. Glucophage is a biguanide used in the treatment of diabetes mellitus, not diabetes insipidus.

A 28-year-old man presents with excessive daytime sleepiness and episodes of cataplexy. He has been having disturbed sleep at night, including dreams so lifelike that they seem like reality and fleeting inability to move when falling asleep. He does not snore. His uncle also has a similar problem. On examination, his weight is 190 lb and his height is 5'8"; his pulse is 72/min and his BP is 130/84 mm Hg. Question What is the most useful non-pharmacologic management of this condition? 1 Weight loss 2 Continuous positive airway pressure 3 Modafinil 4 Schedule routine napping 5 Avoid intake of stimulants before going to bed

Answer: 4 schedule routine napping The correct answer is schedule routine napping. The history suggests the diagnosis of narcolepsy. Besides the daytime sleepiness and cataplexy, the patient is experiencing other symptoms of narcolepsy: hypnagogic hallucinations and sleep paralysis. The scheduling of short naps (15-20 minutes) 2-3 times per day can help control excessive daytime sleepiness and improve alertness. This is the best non-pharmacological measure for this condition. Modafinil is the first-line medication for the treatment of narcolepsy. It has a low abuse potential, and, unlike other stimulants, it is not associated with rebound hypersomnolence. The exact mechanism of action of modafinil is unknown. Modafinil may indirectly increase wakefulness partly through inhibition of GABA release via serotonergic mechanisms, or indirectly on dopaminergic stimulation. It also stimulates norepinephrine inhibition of the sleep-promoting nucleus, the ventrolateral preoptic nucleus. Usual doses of modafinil are 200 mg/d or 400 mg/d, but higher doses may be required in some patients. Note that the question specifically asks for non-pharmacologic treatment. Weight loss and continuous positive airway pressure (CPAP) are non-pharmacologic measures for treatment of obstructive sleep apnea. The patient does not have symptoms of obstructed breathing, the most notable of which is nighttime snoring, making this diagnosis less likely. Avoiding intake of stimulants before going to bed may improve nighttime sleep, but it is not a specific treatment for the daytime sleepiness due to narcolepsy.

A 56-year-old Asian woman presents to establish care for a wellness exam. She recently relocated from out of state; she brings a copy of her medical notes and labs from her previous medical provider. The patient tells you that sexual intercourse is very uncomfortable for her. She admits insertional dyspareunia and poor lubrication, but she denies deep dyspareunia. Outside of intercourse, she denies pelvic issues. She denies vaginal bleeding, pruritus, or discharge. She also denies urinary issues, such as dysuria, frequency, and incontinence. She has occasional hot flashes, but she feels they are tolerable. Her relationship is otherwise healthy, and she does not feel pressured into intercourse against her will. She is a G2P1. Surgical history includes open fracture reduction of the left ankle at age 22 and a dilatation and curettage (D&C) at age 24 for a spontaneous abortion (SAB). She is divorced, and she has been in her current monogamous relationship for 1.5 years. She is a non-drinker and non-smoker; she denies recreational drug use. Family history is unremarkable. The table indicates laboratory tests that were done approximately 1 year ago. CBC Normal TSH Normal FSH Elevated LH Elevated Estradiol Low hCG Negative Chlamydia Negative gonorrhea Negative HIV Negative RPR/VDRL Negative Hepatitis panel Negative Question Based on this patient's history and lab results, what physical exam finding do you expect? 1 Thick white discharge adhering to vaginal mucosa 2 Uterine enlargement 3 Uterine prolapse 4 Vulvar and vaginal atrophy 5 Yellow-green vaginal discharge

Answer: 4 vulvar and vaginal atrophy This patient would be expected to have vulvar and vaginal atrophy on her physical exam. Vulvar and vaginal atrophy occurs after menopause when the estrogen receptors in the epithelium are no longer stimulated as a result of declining ovarian output of sex steroids. With vulvar and vaginal atrophy, the clinician will observe thinning of the labia, less rugation of the vaginal walls, and pale coloration with dry (and possibly friable) vaginal mucosa. This condition occurs gradually after menopause, and patients with this condition may be reluctant to discuss it. Common complaints are vaginal dryness, irritation, dyspareunia, and inadequate lubrication with sexual activity. While routine testing for menopause is not recommended, this patient's laboratory results show an elevated follicle-stimulating hormone (FSH) and luteinizing hormone (LH), with low estradiol, which is consistent with her status as postmenopausal. A thick white discharge adhering to the vaginal mucosa would be the physical exam finding expected in a woman with a vaginal yeast infection (candidiasis). Patients might report vaginal irritation, pruritus, and increased discharge. She may also report insertional dyspareunia. This patient denied the other symptoms and is clearly postmenopausal, putting her at risk for atrophy. Uterine enlargement can be seen in women with uterine fibroids (leiomyomas) or simply after having multiple pregnancies. Uterine fibroids tend to diminish in size after menopause and this patient only experienced two pregnancies. She would not be expected to have appreciable uterine enlargement based on her history and labs. Uterine prolapse may occur in younger women, but it is most common in postmenopausal women. The pelvic floor support and uterine ligaments are inadequate to keep the uterus in place and it descends into the vagina. Prolapse can vary in severity. When it is present, the patient may describe a feeling of vaginal fullness or that "something is falling out." Prolapse is not associated with insertional dyspareunia or poor vaginal lubrication. Yellow-green vaginal discharge indicates vaginal infection. This patient has been tested for sexually transmitted diseases with her current partner, and she currently denies vaginal discharge and pruritus. With her condition of atrophy, the physical exam will reveal less-than-normal vaginal discharge.

A 48-year-old Caucasian woman presents due to feeling like she is losing her mind. She wants some tests done. Upon further questioning, she reports she is having multiple episodes daily in which she suddenly becomes very hot, flushed, and diaphoretic. These episodes last about 1 minute, then resolve. She has not measured a fever. The patient reports that these episodes occur during the day and at night, the latter causing her to awaken drenched in sweat. As a result, her sleep has been poor, and she feels fatigued and irritable both at work and at home. She has noticed these symptoms for about the last 2 months, and they seem to be increasing in severity. The patient has not had a period for 3 months; she recently did a home pregnancy test, which was negative. Prior to that, she had regular menses. This patient denies weight changes, palpitations, cold intolerance, bowel changes, as well as changes in her nails, skin, and hair. Although she admits irritability, she denies anxiety, depressed mood, and suicidal ideation. Her past medical history is significant for seasonal allergies, which are relieved with over-the-counter antihistamines and taken as needed. Her surgical history includes a tonsillectomy and bilateral tubal ligation. Her family history is remarkable for diabetes in her maternal grandfather and hypertension in her father. She is a G4P3Ab1. Social history reveals the patient is an office manager for a dental clinic; she is a married nonsmoker with 3 children living at home. She denies any major psychosocial stressors recently. She drinks alcohol rarely, and she denies use of other drugs. Vitals and a urine specimen for hCG were obtained prior to the physical exam. Weight 168 lb Height 65" Pulse 72 Blood pressure 120/82 Temperature 98.2°F Urine hCG Negative Complete screening physical exam is normal, with normal sexual development and absence of hirsutism and acne. After discussing the likely diagnosis with you, the patient decides she does not want any tests now and wants to start treatment immediately. What medicine should be initiated? 1. Alprazolam (Xanax) 0.5 mg by mouth 3 times daily as needed 2. Clonidine (Catapres) 0.1 mg by mouth 2 times daily 3. Estradiol 1 mg by mouth daily 4. Estradiol/norethindrone acetate (Activella) 1 mg/0.5 mg by mouth daily 5. Estradiol vaginal (Estrace Vaginal) 0.01% Cream 1 g vaginally twice weekly

Answer: 4. Estradiol/norethindrone acetate (Activella) 1 mg/0.5 mg by mouth daily The most effective treatment for vasomotor symptoms in perimenopause and menopause is hormone therapy. Because this patient still has her uterus, a combination of an estrogen plus a progestin should be selected. Estradiol/norethindrone acetate (Activella) 1 mg/0.5 mg daily is the most appropriate choice listed. This patient's history reveals no contraindications to the use of hormone therapy. Alprazolam would be given for panic attacks. This patient's described episodes are consistent with hot flashes and night sweats, and she denies anxiety. Alprazolam will not effectively treat vasomotor symptoms. Clonidine is an alpha-2 receptor stimulator and is used primarily for treatment of hypertension. It has mild efficacy for vasomotor symptoms and may be reasonable if there are contraindications to the use of the more effective hormone therapy. Estradiol may alleviate this patient's vasomotor symptoms, but without an "opposing" progesterone, it is contraindicated. Estrogens by themselves can overstimulate the endometrial lining, leading to endometrial hyperplasia and cancer. Vaginal use of estradiol, such as in Estrace Vaginal, is useful for treatment of vulvar and vaginal atrophy associated with menopause, but the majority of the effect is local with topical use of vaginal cream. This choice would not be effective in alleviating this patient's hot flashes and night sweats. She does not have symptoms or a physical exam consistent with vaginal atrophy (vaginal dryness/dyspareunia or pale, dry mucosa, respectively).

A 33-year-old man presents for an initial visit to a new primary care office. He has not seen a healthcare provider at all in the past 5 years. His past medical history reveals a coarctation of the aorta repair at age 13, after which he saw a cardiologist yearly until age 18. Since then, he has not had insurance and has only sought care for urgent problems in acute care clinics. Question: What is a common complication of coarctation that primary care should regularly monitor this patient? 1. Anemia 2 Palpitations 3 Orthostatic hypotension 4 Hypertension 5 Hypertriglyceridemia

Answer: 4. Hypertension Explanation: Approximately 25% of surgically corrected coarctation of the aorta patients have persistent or recurrent hypertension years after surgery because of changes in the renin-angiotensin system. They often have disproportionate systolic hypertension with exercise. These patients are also at risk for re-coarctation, aortic aneurysm (especially near the repair site), and left ventricular hypertrophy. Coarctation patients need to have their blood pressure monitored regularly; if they develop hypertension, it should be aggressively controlled. It is also best to have these patients see a cardiologist. Anemia, palpitations, orthostatic hypotension, and hypertriglyceridemia are not associated with coarctation of the aorta. Some studies have found hyperlipidemia more commonly in patients with coarctation, but not hypertriglyceridemia.

An 83-year-old man presents to his urologist's office with "problems down there." He is a poor historian, but his wife accompanies him and helps with details of the history of present illness. She does not report that he has any voiding issues and reports that he has never had to see a urologist before now. Further genitourinary review of systems questions is suggestive of phimosis. Question What physical examination finding would support this diagnosis? 1 Entrapment of the foreskin behind the glans penis in an uncircumcised male patient 2 Erythema and edema of the glans penis in an uncircumcised male patient 3 Erythema and edema of the phallus in a circumcised male patient 4 Erythema and edema of the glans penis in a circumcised male patient 5 Inability to retract the foreskin over the glans penis in an uncircumcised

Answer: 5 Inability to retract the foreskin over the glans penis in an uncircumcised Inability to retract the foreskin over the glans penis in an uncircumcised male patient is the correct answer. Phimosis can be the result of recurrent infections or irritation, advanced age, diabetes, and poor hygiene. Patients can experience painful erections, recurrent balanitis, and voiding difficulties. Treatment can be with topical steroids followed by gradual retraction of the foreskin or circumcision. Entrapment of the foreskin behind the glans penis in an uncircumcised male patient describes a patient with paraphimosis. Typically, this results in retraction of the foreskin for medical or hygiene purposes that is not followed by properly pulling the foreskin back in place over the glans penis. This can be very painful and needs to be manually reduced as soon as possible to prevent necrosis of the glans penis. A dorsal slit or circumcision may be necessary if manual reduction is not possible. Erythema and edema of the glans penis in an uncircumcised male patient most closely describes balanitis and does not address the placement or status of the foreskin, which is the main concern with phimosis. Erythema and edema of the phallus/glans penis in a circumcised male patient are not correct, as patients who are circumcised cannot experience phimosis due to their lack of foreskin.

An 18-year-old woman presents with a 1-week history of vaginal discharge that has become progressively worse. She reports recent unprotected intercourse. On exam, the white discharge appears watery and she exhibits cervical motion tenderness. A KOH wet prep is negative without clue cells or hyphae. Question What is the most appropriate treatment? 1 Fluconazole 2 Penicillin G 3 Metronidazole 4 Ceftriaxone 5 Azithromycin

Answer: 5 azithromycin The clinical picture is suggestive of a chlamydia infection; the most common organism is Chlamydia trachomatis. While all of the answer choices are utilized to treat gynecologic infections, the recommended treatment for chlamydia is a single oral 1 g dose of azithromycin. Other treatment options include doxycycline or levofloxacin. None of the other options are appropriate treatment options for chlamydia. Ceftriaxone is the preferred treatment for gonorrheal infections. Fluconazole treats candidiasis or yeast infections. Metronidazole is the treatment for bacterial vaginosis and trichomoniasis. Penicillin G is utilized to treat syphilis.

A 3-month-old male infant has been brought into the pediatric clinic for assessment. The newborn's mother states that her child is not gaining adequate weight despite a regular breastfeeding schedule. She additionally has noted that the child appears to get "very tired and inactive" during and after feedings and that she is able to feel copious amounts of sweat on the child's skin following feedings. She denies any known illness in her child and recalls a normal birth. The general survey reveals a weight and length in the 40th percentile, tachycardia, and tachypnea. The cardiac exam is remarkable for a bounding and hyperdynamic precordium, a holosystolic harsh murmur that is audible over the lower sternal border (LSB), and a loud second component of the second heart sound. Question What is the best diagnostic tool to confirm the diagnosis? 1 Chest radiograph 2 Electrocardiogram 3 Cardiac catheterization 4 Magnetic resonance imaging 5 Echocardiogram

Answer: 5 echocardiogram Chest radiography, magnetic resonance imaging (MRI), and electrocardiography (ECG) may all provide useful information in the workup of a ventricular septal defect (VSD). Detailed echocardiography is now preferred in most institutions. Echocardiography provides the information required for potential surgical closure of a VSD. On chest radiograph (CXR), there is moderate to marked enlargement of the heart, with prominence of the main pulmonary arterial segment and overcirculation in the peripheral lung fields. The left atrium is dilated. While this may be an initial test, echocardiography is the best test to confirm the diagnosis of VSD. On electrocardiogram, left ventricular hypertrophy, right ventricular hypertrophy, and right atrial enlargement are all possible in VSD. In patients with moderate-sized VSDs and with moderate or large left-to-right shunts with volume overload in the left ventricle (LV), LV hypertrophy is the rule. Combined ventricular hypertrophy is common. This may manifest as a large equiphasic mid-precordial voltage (>50 mm) in the mid-precordial leads, an event known as the Katz-Wachtel phenomenon. Inlet defects may be associated with left-axis deviation of the frontal plane QRS with Q waves in leads I and aVL. In patients with large VSDs and equal ventricular pressures, RV hypertrophy is demonstrated. While electrocardiogram may be an initial test in the diagnosis of VSD, echocardiography is the best test to confirm the diagnosis. Cardiac catheterization was a standard part of the evaluation of a VSD in the past. Cardiac catheterization is used primarily in the following 2 settings: Pulmonary hypertension of unknown reactivity A small-to-moderate defect with only mild left ventricular (LV) enlargement; in this setting, cardiac catheterization is useful for definitively assessing the pulmonary-to-systemic flow ratio (Qp:Qs), which can assist decision making regarding the need for surgery An MRI can provide this information noninvasively. MRI data about systemic and pulmonary flows have been well validated and well correlated with catheterization data, and one of the indications for the use of MRI is evaluation of a VSD that is judged to be borderline during echocardiography in terms of the level of the left-to-right shunt. For such defects, an MRI-derived Qp:Qs may assist the clinician in making the decision whether to proceed with surgical treatment. Two-dimensional imaging can distinguish an uncomplicated VSD from more complex malformations and is capable of imaging most defects directly when multiple transducer positions are used. The addition of pulsed-wave Doppler with color-flow mapping permits the identification of small, multiple, muscular, and other less easily visualized defects. The position and size of the VSD can be determined, as well as its relationships to the aorta, pulmonary artery, and AV valves.

A 33-year-old woman presents for an annual physical. She has past medical history of GERD, asthma, and irritable bowel syndrome. She drinks 1-2 alcoholic beverages per week and has never smoked; she does not use illicit drugs, and she consumes a vegetarian diet. Her past surgical history includes an appendectomy at age 14. Her father passed away from a heart attack at age 63. Her mother is alive with history of colorectal cancer, which was diagnosed at age 41. Question What is the recommended colorectal cancer screening for this patient? 1 Screening colonoscopy beginning at age 50 2 Screening colonoscopy beginning at age 45 3 Screening colonoscopy beginning at age 40 4 Screening colonoscopy if the patient becomes hemoccult positive 5 Immediate screening colonoscopy

Answer: 5 immediate screening colonoscopy The above patient should have an immediate screening colonoscopy. The American College of Gastroenterology currently recommends that patients with positive family history of colorectal cancer should begin screening colonoscopy at either age 40 or 10 years younger than the age at diagnosis of their relative. The patient's mother was 41 years old when she was diagnosed with colorectal cancer, which means that the patient should have undergone her first screening colonoscopy at age 31. Since the patient is currently 33 years old, she should undergo colonoscopy as soon as possible. Screening colonoscopy beginning at age 40, 45, and 50 are incorrect because of the age the patient's mother was when she was diagnosed with colorectal cancer. Screening colonoscopy if the patient becomes hemoccult positive is an incorrect answer. Screening should begin immediately; the primary care physician should not wait for signs of colorectal cancer to develop prior to screening.

48-year-old African American woman presents with a concern about osteoporosis. Her elderly father-in-law recently suffered a fractured hip due to severe osteoporosis. The patient has been having intermittent bilateral hip pains and is worried that she also has osteoporosis. She gets the hips pains more with prolonged standing, walking, and heavy lifting. At today's exam, she reports the pain as mild (1 out of 10, on 1-10 scale). A summary of her past medical history is shown in the table: Medications Fluticasone/salmeterol (Advair HFA) inhaler daily for asthma control; ibuprofen 400 mg as needed for arthritis pain Medical history Asthma, mild osteoarthritis OB/GYN history G2P2, has regular menses Social history Patient admits to tobacco use (1 pack of cigarettes daily). She denies use of alcohol and illicit drugs. She is divorced with 2 children at home. She walks for exercise and works in a potato processing facility, which involves frequent lifting of moderate weight. Family history Positive for breast cancer in patient's mother; hypertension in her father; asthma in 1 brother Question What aspect of this patient's history puts her most at risk for development of osteoporosis? 1 Age 2 Current symptoms 3 Ethnicity 4 Family history 5 Social history

Answer: 5 social history The patient's social history is the most worrisome risk factor for development of osteoporosis. Tobacco use conveys a greater risk of developing osteoporosis. The patient's age is not a risk factor for osteoporosis. Increased age (and postmenopausal status for women) is associated with increased risk of osteoporosis. The patient's current symptoms of mild hip pain are not associated with osteoporosis. Typically, patients with osteoporosis are asymptomatic, even in fairly severe cases. Ethnic and racial groups shown to be highest risk for osteoporosis are Caucasians, Asians, and Native Americans. African Americans seem to have a lower risk; research has not yet determined the reason. The patient's family history is negative for osteoporosis. The other conditions in her family history are not linked with heritable osteoporosis risk. The father-in-law with osteoporosis does not confer any genetic risk to this patient.

37-year-old Hispanic man presents with a 4-month history of mild-to-moderate headaches; on average, he gets them 3-4 days per week. He has tried over-the-counter analgesics with minimal relief. He is seeking care now because he had been promoted to store manager several months prior to presentation, and he is worried that his headaches are affecting his concentration. His headaches are generalized in location, described as starting at the base of his head and extending all over, feeling "tight" in nature. He denies memory loss, photophobia, nausea/vomiting, rhinorrhea, lacrimation, and upper respiratory symptoms associated with the headaches. He also denies seizures, syncope, incoordination, vertigo, weakness, and paresthesias. The patient mentions his concerns for his work several times. Although he enjoys his work, he admits to having some anxiety about being able to handle his new duties. His family history is negative for headache. Physical exam is performed; vitals, neurological, cardiovascular, and HEENT findings are all normal. Question What is the most likely diagnosis? 1 Cluster headache 2 Headache secondary to brain tumor 3 Migraine headache 4 Subarachnoid hemorrhage 5 Tension headache

Answer: 5 tension headache This patient presents with a most likely diagnosis of tension headache. Tension headaches, the most common type of headache, present as rather diffuse/generalized head pain, without the classic characteristics of the other types of headaches. A component of psychological stress is often present. Cluster headaches are a severe type of headache, most often occurring in middle-aged men, in which the severe headache is accompanied by several other symptoms, such as lacrimation, nasal rhinorrhea, and congestion. The pain is usually localized in the periorbital region. About 1/3 of patients with a brain tumor will present with a headache, but tension-type headaches are far more common in the general population. Headaches secondary to brain tumor can vary in character and intensity. New onset headaches later in life, especially those without a coinciding significant psychosocial stressor, are of greater concern; they have a larger possibility of being caused by an intracranial mass. Nausea and vomiting or neurologic symptoms may accompany headaches caused by a brain tumor. Migraine headaches are also quite common. They are classically described as throbbing in nature and are accompanied by nausea (and possibly vomiting), photophobia (sensitivity to light), and phonophobia (sensitivity to sounds); visual changes and aura are also possible. Migraines are more common in women than men, but they can occur in men. They are usually a chronic condition, beginning in adolescence or the early adult years. Stress can be among many triggers for migraines. This patient's pain pattern and age of onset are inconsistent with migraine, and he lacks the associated symptoms. Subarachnoid hemorrhage (SAH) is classically described by patients as an acute headache that is the "worst headache of my life." There may be preceding trauma and associated nausea and vomiting; nuchal rigidity (due to meningeal irritation) and mental status changes are also seen. This patient's headache history is inconsistent with SAH.

A 24-year-old man presents due to increasing pain in his right groin and buttock and difficulty walking. The pain has been present and worsening for about 1 month. He further reports that he had quite significantly injured himself also near his right hip in a fall while snowboarding about 3 months ago. He did not seek care immediately and had difficulty ambulating for a week, then saw a chiropractor to "put his lower back back in." The pain and gait problems eventually improved. Now he is concerned because the pain has returned with no new history of trauma. He is otherwise healthy and takes no regular medicines. He denies any chronic health conditions and has an unremarkable family history for musculoskeletal and rheumatologic conditions. On physical exam, the patient was noted to walk into the exam room with a slight limp. Examination of the hip, buttock, and groin region reveals no edema, erythema, or ecchymosis. The exam does not produce one particular point of tenderness with palpation, but pain is elicited with right hip motions, particularly internal rotation. Diagnostic testing revealed a "crescent sign" and confirmed the suspected condition. The patient will be scheduled for definitive treatment within a week. Question What approach is best to prevent further complications and maintain his current health until definitive treatment occurs? 1 Begin aggressive physical therapy. 2 Initiate antibiotics. 3 Initiate bisphosphonate therapy. 4 Initiate high-dose corticosteroids. 5 Utilize crutches for any ambulation.

Answer: 5 utilize crutches for any ambulation This patient's condition is suspicious for avascular necrosis (or the more preferred term, osteonecrosis) of the hip. Avascular necrosis (AVN) is a relatively common complication following a traumatic hip dislocation when the vascular supply to the femoral head is damaged and bone death occurs. There are many theories about the exact mechanism; other causes, such as vascular, congenital, and autoimmune diseases, have been implicated with osteonecrosis. If the "crescent sign" is seen on plain radiography (X-ray), AVN is confirmed. Definitive treatment is surgical, with a variety of approaches, from core decompression to bone grafting. Until surgery occurs, symptomatic patients should utilize crutches for any ambulation to prevent collapse of the femoral head. The patient should not begin aggressive physical therapy prior to surgical repair. As noted above, bearing weight on the joint may predispose the patient to bone collapse. There is also no reason to initiate antibiotics prior to surgical treatment. AVN is not caused by infection, in contrast to osteomyelitis. Likewise, the provider should not initiate bisphosphonate therapy, even though the long-term goal is for the patient to regrow new bone. In fact, bisphosphonate therapy has been linked with causing osteonecrosis. There are some studies proposing benefit with using bisphosphonates, but not enough definitive data yet to recommend their use. The provider should not initiate high-dose corticosteroids, either. As with bisphosphonates, corticosteroids have been linked with causation of osteonecrosis. The primary mechanism of AVN does not appear to be inflammatory, so the steroids would not benefit the patient's healing.

While performing a comprehensive oral exam on a patient, you note 3 lesions on the buccal mucosa on the right side of the oral cavity. You document these lesions with the following description: 3 round lesions, each measuring approximately 2 mm in diameter, with the presence of a white-yellow center surrounded by a red halo. When questioned, the patient states that this is "nothing new" and that they usually appear during her period. Question What clinical intervention should you recommend to this patient to help aid in healing? 1 Oral antivirals 2 Topical antibiotics 3 Topical corticosteroids 4 Oral aspirin 5 Oral acetaminophen

Answer: topical corticosteroids This patient has an episode of recurrent aphthous stomatitis (RAS); they are also sometimes referred to as canker sores. This is a common inflammatory condition of the oral cavity often starting in childhood; they are typically small, recurrent, painful, round or ovoid ulcers with well-defined erythematous margins. These erythematous margins appear as a halo with a central yellow or gray floor. They are benign, non-cancerous, non-infectious, and in most cases, the cause is unknown. The lesions are often described as having a white or yellow center, and they are surrounded by a bright red area. They are occasionally related to the menstrual cycle. Topical corticosteroids remain the mainstay of therapy for RAS. Although they reduce the painful symptoms, they do not decrease the recurrence of symptoms. Popular preparations for the treatment are hydrocortisone and triamcinolone because they do not cause significant adrenal suppression. Topical antibiotics and oral antivirals are not indicated at this time; there is no direct evidence of a virus or bacteria as the etiology of aphthous ulcers. Aspirin and acetaminophen do not have any significant effects in terms of helping heal aphthous ulcers. These agents may aid in relieving some of the pain associated with them, but they will not provide any aid in healing.


Set pelajaran terkait

The Internet, Chapter 7, "The Public Sphere"

View Set

Nursing Health Alterations Exam 3

View Set

HA Prep U: Chapter 3: Collecting Objective Data: The Physical Examination

View Set

CH1 The Nature and Importance of Leadership

View Set

CHAPTER 22 LESSON 3: HIGH-FRUCTOSE CORN SYRUP AND NONNUTRITIVE SWEETENERS

View Set

Chapter 11 - 12 Homework- DNA Profiling✅✅

View Set

Chem 1212 - 17.2 Factors Affecting Reaction Rates

View Set

Saunders ch30 Postpartum Complications

View Set